You are on page 1of 117

QUESTION BANK

1

Match the column type questions
1. For a particle moving along a circle of radius r, at a given instant of time
List I List II
I) Centripetal acceleration = 0 implies that A) a
c
= 0
II) Tangential acceleration = 0, implies that B) a
T
= 0
III) If speed is increasing C) angle between velocity and

a is obtuse
IV) If the speed is decreasing D) angle between

C
a and

a is acute
A) I-A, II-B, III-D, IV-CD B) I-A, II-C, III-D, IV-B
C) I-B, II-C, III-A, IV-D D) I-D, II-A, III-B, IV-C

2. A particle is performing circular motion. Then if
List I List II
I) It has a constant speed, then A) It cant follow a circular path
II) It has a varying speed, then B) It can have varying magnitude of acceleration
III) It has varying velocity, then C) It has constant magnitude of acceleration
directed towards the centre.
IV) It has a constant velocity, then D)Its acceleration also has a tangential component
A) I-A, II-D, III-C, IV-A B) I-A, II-B, III-D, IV-A
C) I-D, II-C, III-A, IV-B D) I-D, II-A, III-B, IV-C

3. Two particles A and B are moving in 2concentric circles of radii r
A
and r
B
respectively in same plane. The
angular velocity of A is
A
e and that of B is
B
e . At t = 0, both are in same phase, then if
List I List II
I) e e e = =
B A
and 2r
A
= r
B
=r, then relative A) 2 / 5e
angular velocity of B w.r.t. A at t = 0, is
II) e e e = =
B A
2 and r
A
= 2r
B
=r, then relative B) 2 / e
angular velocity of B w.r.t. A at t = 0, is
III) e e e = =
B A
3 and 3r
A
= r
B
=r, then relative C) zero
angular velocity of B w.r.t. A at t = 0, is
IV) e e e = = 2 /
B A
and r
A
= (r
B
/3 )=r, then relative D) 2 / e
angular velocity of B w.r.t. A at t = 0, is
A) I-B, II-D, III-C, IV-A B) I-A, II-B, III-C, IV-D
QUESTION BANK
2
C) I-C, II-A, III-B, IV-D D) I-A, II-D, III-C, IV-B



4. A particle is moving along a parabolic path y = x
2
. At the instant when the particle is at (1, 1).

List I List II
I) a
c
of the particle
t
a s m
2
/ 2 2 = of the particle = 1 m/s
2
,
then acceleration of particle in ( ) u , r coordinate is A)
( ) ( )
^ ^
10
9 7
10
1 7 3
u
+
+

r
II) a
c
and
2 2
/ 1 / 2 2 s m a and s m a
t c
= = then acceleration
of particle in (x-y) coordinate is B)
( )
^ ^
5
6
10
1
5 2
4 2 3
u
|
|
.
|

\
|
+ +

r

III)
2 2
/ 7 / 3 s m a and s m a
t c
= = , then acceleration of
particle in ( ) u , r coordinate is C)
( )
( )
^ ^
5
2 4 1
2 1
5
2
i j

+ +

IV)
2 2
/ 7 / 3 s m a and s m a
t c
= = , then acceleration of
particle in (x y) coordinate is D)
( ) ( )
^ ^
5
6 7
5
3 7 2
i j

+
+

A) I-B, II-C, III-A, IV-D B) I-A, II-B, III-C, IV-D
C) I-C, II-A, III-B, IV-D D) I-A, II-D, III-C, IV-B

5. A particle is following a path x
2
= y(2 y) (x, y > 0) and y(t) =t
2
. Now at t = 1s.
List I List II
I) x coordinate of the particle is A) 6
II) magnitude of a
c
of the particle is B) 1
III) radius of curvature is C) 5
IV) speed of the particle is D) 5 / 6
A) I-A, II-B, III-C, IV-D B) I-B, II-A, III-D, IV-C
QUESTION BANK
3
C) I-C, II-A, III-B, IV-D D) I-A, II-D, III-C, IV-B

6. A particle is moving in circle in x-y plane of equation x
2
+ y
2
= 1 with an angular velocity ' 'e , if
List I List II
I)
( )
( ) ( ) 0 , cot cos
0 0
> = V V V
y
t and the particle
covers complete circle in 2 / t sec, then A) at
2
1
,
2
1
= = y x
II) ( ) ( ) 0 , cot cos
0 0
< + = V V V
y
t , and ' 'e is 8 rad/sec, then B) at time
x
V sec, 8 / t will be V
0

III) ( ) ( ) 0 , 2 / sin
0 0
> + = V t V V
y
t e and t e 2 = t
at t sec, 4 / t = t then C) at time
x
V sec, 16 / t will be V
0

IV) ( ) ( ) 0 , 4 / cos
0 0
> + = V t V V
y
t e , if sec / 4rad = e D) at
x
V y x ,
2
1
,
2
1
= = will be
2 /
0
V
A) I-A, II-D, III-C, IV-B B) I-A, II-B, III-C, IV-D
C) I-C, II-A, III-B, IV-D D) I-B, II-C, III-B, IV-AD

7. A particle is performing circular motion such that ( ) 0 > e . The a
c
vs t and t s v/ e graphs for the motion are
plotted in left column
List I List II
I) A) a
T
< 0

II) B) a
T
= 0

QUESTION BANK
4
III) C) particle is speeding up

IV) D) particle is speeding down

8. A particle is moving along a circle x
2
+ y
2
= a
2
in horizontal plane with angular velocity =
2rad/sec. A man is moving along the x = 2a in + direction with velocity 4a m/s in the same plane. At t = 0,
particle and man are along line y = -a
List I List II
I) At 2 / t = t , the angle made by the line of motion of
particle with line parallel to x-axis w.r.t. man is A) ( ) 2 2 1 tan
1
+ +


II) At 8 / t = t the angle made by the line of motion of
particle with line parallel to x-axis w.r.t. man is B) a j i
|
.
|

\
|
+
^ ^
4 2 2
III) At 4 / 3t = t , the velocity of particle w.r.t. man is C) 2 / t
IV) At , the angle made by the line of motion of particle
with the line parallel to x-axis w.r.t. man is D)
^ ^
4 2 j a i a +
A) I-A, II-B, III-C, IV-D B) I-B, II-C, III-A, IV-A
C) I-C, II-A, III-B, IV-D D) I-A, II-D, III-C, IV-B

9. A particle moving in horizontal plane along a curve whose equation in polar coordinates is u cos 2a r =
List I List II
I) If , 60 & / 5 /
0
= = u u s rad dt d then A) ( )
2
/ 2 1
3
100
s m
a
a
N
+ =
II) If V = 10a m/s and , 30
0
= u then B)
2
/
3
50
s m
a
a
T
=
III) If
0
2
2
45
3
25
= = u
u
and
dt
d
, then C)
2
/ 100 s m a a
N
=
QUESTION BANK
5
IV) If , 2 / sec / 10 rad and rad t u e = = then D)
2
/
3 2
50
s m
A
a
T
+
=
E)
2
/ 0 s m a
T
=
A) I-A, II-B, III-C, IV-D B) I-CD, II-BC, III-AB, IV-CE
C) I-C, II-A, III-B, IV-D D) I-A, II-D, III-C, IV-B

10. A massless ring is rolling on frictionless surface with velocity V and angular velocity ' 'e such that V =


List I List II
I) Acceleration vectors of points A & B are A) Directed towards centre
II) Velocity vectors of points A & B are B) perpendicular to each other.
III) Acceleration vectors of points B & C are C) at angle 45
0
with each other.
IV) Velocity vectors of points B & C are D) in opposite directions
A) I-C, II-C, III-C, IV-A B) I-A, II-B, III-C, IV-D
C) I-AB, II-C, III-AD, IV-B D) I-A, II-D, III-C, IV-B

Kinematics-I
11. A drunkard man moves 5m east 10m north and then 20m southwest. Find out the total distance and
displacement traced.

12. A wheel is rolling on horizontal surface. Find out the displacement of a point mark at the bottom most point on
wheel of radius R which rolls half revolution.


13. In the above example consider the point on the extreme right. Get the disp. for half revolution.
QUESTION BANK
6


14. In a circular motion, plot the graph of displacement vs angle u .


15. A train moves a distance s with speed v
1
and comes back along same path same distance s with speed v
2
. Find
out average speed and average velocity.

16. If the train moves with speed v
1
for first half time and speed v
2
for second half-time. Find out the average
displacement.

17. A particle has initial velocity = 10m/s east, acceleration = 5m/s
2
is given in north direction. Find out the final
velocity and displacement at t = 2 sec and direction.

18. A train starts form rest with acceleration = a, moves uniformly and then retards with retardation b, then comes
to rest. If total displacement travelled is s, find the minimum time taken to cover the distance.

19. A balloon starts rising up with an acceleration a = 5m/s
2
, at t = 4 sec. the stone is de-attached, which first rises
up and then comes down. Find out the total time taken of stone before hitting the ground. Plot the displacement
vs. time graph.

20. Consider the disp. X as a function of time t. x = 4t t
2


21. Consider the potition x as a function of t. x = (t -1)(t -2)(t-3)

22. The particle A is moving with constant velocity u and B approaches A with a velocity v, initial separation
being l.
QUESTION BANK
7


23. A man of height 1.5m moves away from a bulb 6m high with speed 2m/s. Find out velocity of shadow.


24. A stone is dropped form the top of a t tower of a unknown height h. At the instant it has fallen c metres, a
second stone is released from rest from a point b m below the top of tower (b > c). The two stones strike the
ground at the same time. Determine the height of tower.


25. A stone is dropped in air at height 39m above ground, due to wind blowing it has horizontal acc. a
x
=
1.2m/s
2
. Describe its path and find out range.

26. Two cars are moving with velocities v
1
and v
2
towards the crossing (v
1
> v
2
). Taking their initial separation
from crossing being a each. Find out the relative velocity of car B w.r.t. A and min. separation between them.

27. A motor boat going downstream overcame a raft at point A. After one hour it turned back and met the raft
again at a distance 6km from point A. Find the river velocity.

28. Two boys at the bank of river started swimming simultaneously with same speed w.r.t water, one swims
perpendicular to the stream and other aims to do so. Other boy deflected by some distance from the destination
now he walks to the destination. If both reach the destination at the same time, find out speed of walking of
other boy. (Take velocity of flow = 3, velocity of each boy in still water = 5 km/hr)
QUESTION BANK
8

29. A balloon starts rising from the earths surface. The accession rate is constant and equal to v
0
. Due to the wind,
the balloon gathers the horizontal velocity component v
x
= ky where k is a constant any is a height of ascent.
Find how the following quantities depend on the height of ascent.
a) The horizontal drift of the balloon x(y).
b) The total tangential and normal accelerations of the balloon.

30. Two trains initially separated by distance L are heading towards each other on same track each with speed v,
and a bird flies form train A towards B with constant speed w > v reaches train B and immediately comes back
to A with same speed and continues to do so till it sandwiches between the two. Find out the number of trips
and time taken before it sandwiches. Show the situation graphically.

31. A car is moving with constant velocity 25m/s ahead 100m of a jeep starts accelerating a = 2m/s
2
. Show that the
two values of time the car and jeep are at the same position and show the situation graphically.


MCQS with one correct answer
32. A particle moves according to eq. t = ax
2
+bx, find the retardation at any instant.
a)
( )
3
2
2
b ax
a
+
b) 2a c)
( )
2
2
2
b ax
a
+
d) None of these

33. A particle falls from a height h. In the last 0.2sec it travels 6m. Find the height h.
a) 48.05m b) 420.05m c) 32.05m d) None of these

34. A particle starts with an acc. o from rest for sometime and after achieving max. velocity starts retarding at rate
| and finally comes to rest. If total time taken t, its max. velocity during the motion will be
a)
| o
| o
+
t
b) t
|
.
|

\
| +
2
| o
c) t
| o
| o
+
2
d)
( )
t
| o
| o
2
2
+


35. A water tap leaks such that water drops fall at regular intervals. Tap is fixed 5cm above the ground. First drop
reaches the ground when 4
th
is about to leave the tap. Find the separation between 2
nd
and 3
rd
drop.
a) m
3
2
b) m
3
4
c) m
3
5
d) None of these
QUESTION BANK
9

36. An elevator is descending with uniform acc. To measure the acc. a person in the elevator drops a coin
6ft.above the floor of the elevator. The coin strikes the floor in 1sec. Find the acc. of elevator.
a) 20 ft/s
2
b) 10 ft/s
2
c) 16 ft/s
2
d) None of these

37. A police jeep is chasing with velocity 45km/h. A thief in another jeep is moving with 155km/h. Police fires a
bullet with a muzzle velocity 180m/s. The bullet strikes the jeep of the thief with a velocity
a) 27 m/s b) 150 m/s c) 250 m/s d) 450 m/s

38. A truck is moving with a velocity 36km/h. On seeing red light it decelerates at 2m/s. The reflex time of the
driver is 0.4sec. How much distance will truck travel before coming to a stop?
a) 25 m b) 29 m c) 35 m d) None of these

39. A clock has minute hand of 10cm length. The average velocity of the tip of minute hand form 6am to 6:30am
is
a) 2 / 3 cm/min b) 3 / 2t cm/min c) 3 / t cm/min d) None of these

40. A river is 1km wide and flows at a rate 4km/h. A man is capable to swim 3kw/h. Where will he land if he
strikes normally to the flow of river?
a) 1km just opposite bam b) km
3
4
away c) km
4
3
away d) None of these

41. Rain falling vertically downwards with a velocity of 3km/h. A person moves on a straight line road with a
velocity of 4km/h. Then the apparent velocity of the rain w.r.t. the person is
a) 1 km/h b) 5 km/h c) 4 km/h d) 3 km/h

Objective questions with more than one correct choices
42. Two cars A and B start from rest and move together for the same time interval. Their acceleration graphs are as
follows:

a) Both the cars attain the same velocity after 10sec.
QUESTION BANK
10
b) The velocity of car B is more than that of A after 10sec.
c) The distance travelled by the car B is more than that of A after 10sec.
d) Both the cars travel the same distance after 10 sec.

43. A particle moves 10m in the first 2sec, 20m in the next 3sec and 30m in the next to 10sec. Which of the
following statement(s) is (are) true for this motion?
a) The particle was uniformly accelerated
b) The particle had decelerating forces acting on it part of the motion
c) The average speed of the particle was 4 m/s.
d) The average acceleration of the particle was 3 m/s
2


Comprehension Questions : I
For spiral motion, the angular displacement as well as the radius keep on changing w.r.t. time, if the radius is
defined as a function of angular disp. ( ) u f r = , it changes w.r.t. u . Hence the particle will have tangential
and radial component in polar coordinates.

Radial velocity
dt
dr
t
PM
t
OP OM
= =


o o
u o u o 0 0
lim lim Tangential velocity
dt
d r
t
r
t
QM u
o
u
o
u o u o
= =

sin
lim lim
0 0

44. A particle moves along a path defined by polar coordinates r = 2e
t
and
2
8t = u rad, its tangential velocity at t =
1 sec is
a) 87 m/s. b) 174 m/s c) 43 m/s d) 54 m/s

45. Its radial velocity at t = 1sec is
a) 8.7 m/s b) 87 m/s c) 5.4 m/s d) 54 m/s

Directions. Q: 36 38: In mathematic, the symbol [.] is used for greatest integer which the max. possible value in
integer for that real number e.g., [4.3] = 4 and [-6.3] = -7. Consider a function of acc. as time a = [t-1], if the
body starts from rest t = 0
46. Find out the velocity at t = 3sec.
a) 3 m/s b) 2 m/s c) 1 m/s d) 0 m/s

47. Find out the disp. at t = 3 sec
QUESTION BANK
11
a) 4 m b) 2 m c) 0 m d) -2 m

48. During the motion the body has reversed its direction of motion at
a) t = 1sec. b) t = 2 sec c) t = 3 sec d) t = 1.5 sec

Match the following:
49. List I List II
I) Tangential acc. is zero A) Rectilinear uniform motion
II) Only normal acc. is zero B) Uniform circular motion
III) Resultant acc. is zero C) Non-uniform circular motion.
IV) None of the acc. is zero D) Uniformly accelerated moiton.
a) I-B, II-D, III-C, IV-A b) I-B, II-D, III-A, IV-C
c) I-D, II-C, III-A, IV-B d) I-A, II-B, III-C, IV-D

50. Consider the following graphs, where a = acc., v = vel., s = disp.
List I List II
I) A) Possible
II) B) Possible
III) C) Impossible

IV) D) Impossible

a) I-C, II-B, III-A, IV-D b) I-B, II-C, III-D, IV-A
c) I-C, II-D, III-B, IV-A d) I-A, II-D, III-B, IV-C

QUESTION BANK
12
Numerical Questions
51. Two steel balls fall freely on an elastic slab. The first ball is dropped from a height h
1
and the second from a
height h
2
(h
2
< h
1
), T sec after the first ball. After the passage of time T, the velocities of the balls coincide in
magnitude and direction. Determine the time T and the time interval during which the velocities of the two
balls will be equal assuming that the two balls do not collide.

NEWTONS LAWS OF MOTTION
52. The mass of each ball is m. radius r. and diameter of the cylindrical container is 7r/2. Find out the normal
contact force between the two balls


53. Three identical cylinder are place symmetrical on conjugate inclined plane as shown. Find out the min. value
of angle u for this to be in equilibrium.


54. A heavy rope of mass M, length L is resting on a smooth horizontal surface, it is being pulled by force F. Find
out tension on it at a distance x from the free end.

55. Two blocks of mass 1kg and 2kg are connected by a rubber cord, 2kg block is being pulled with force 10N. If
acc. of this block is 2m/s
2
, find out acc. of other block at that moment.



56. Find out the force exerted by man to keep himself balanced, and normal reaction force between man and plank.
QUESTION BANK
13


57. Check the balance of beam.


58. Find out the free F applied by support and angle | for equilibrium.


59. If a spring (k) is cut into two parts of length ratio n : 1, find out the spring constant of each part

60. A smooth semicircular wire track of radius R is fixed in a vertical plane. One end of a massless spring of
natural length 3R/4 is attached to the lowest point O of the wire track. A small ring of mass m which can slide
on the track is attached to the other end of spring makes an angle 60
0
with the vertical. The spring constant, k =
mg/R. Consider the instant when the ring is released.
a) Draw F.B.D of ring b) Find out tangential acc. of ring and normal reaction on it


QUESTION BANK
14
61. Find out acc. of blocks and force exerted by clamp. (Take, m = 1kg)

62. There is a groove cut on the surface of inclined plane, as shown. Find out the time taken by a block released at
top to travel length, L of groove.


63. Find out the height h of inclined plane if a block placed at top takes minimum time to slide down. Base length
b is constant.


64. Two particles each of mass m are connected by a light string of length 2L as shown. A continuous force F is
applied at the midpoint of the spring (x = 0) at right angles to the initial position of the string. Show that acc. of
m in the direction at right angle to F given by
2 2
2
x L
x
m
F
a
x

= .


65. Suppose a lift accelerating up with acceleration = a. Find out the acc. of each block w.r.t. lift and tension in the
string.


QUESTION BANK
15
MCQs with one correct answer
66. A uniform chain of length L and mass m lies on a smooth horizontal table with its length perpendicular to the
edge of the table and small part overhanging. The chain starts sliding down from rest due to the weight of
hanging part. Find the acceleration and velocity of the chain when length of the hanging portion is x.
a)
L
x g
L
x g
2
, b) x g
L
x g
, c) gL
L
x g
, d) ( ) x L g
L
x g
,

67. If is coefficient of friction between the tyres and road, then the minimum stopping distance for a car of mass
m moving with velocity v is
a) g v b)
g
v
2
2
c) g
2
d)
g
v
2



68. Aballoon of mass M is under a drag force F and upthurst T. It is moving down with a uniform velocity v. What
amount of mass be removed so that it starts rising up with same velocity v?
a) M T/g b) 2T / g c) T / g d) 2(M-T/g)

69. Consider the shown arrangement. Assume all surfaces to be smooth. If N represents magnitudes of normal
reaction between block and wedge, then acceleration of M along horizontal equals

a)
M
N u sin
along +ve x-axis b)
M
N u cos
along +ve x-axis
c)
M
N u sin
along -ve x-axis d)
M m
N
+
u sin
along -ve x-axis

70. In the above problem normal reaction between ground and wedge will have magnitude equal to
a) mg N + u cos b) mg Mg N + + u cos c) Mg N + u cos d) mg Mg N + + u sin

71. A pendulum of mass hangs from a support fixed to a trolley. The direction of the string when the trolley rolls
up a plane of inclination o with acceleration, a, is
QUESTION BANK
16

a) zero b) tan
-1
o c)
|
|
.
|

\
| +

o
o
cos
sin
tan
1
g
g a
d) tan
-1
g
a


72. Two masses m and M are attached with strings as shown. For the system to be in equilibrium, we have

a)
m
M 2
1 tan + = u b)
M
m 2
1 tan + = u c)
m
M
2
1 tan + = u d)
m
M 2
1 tan + = u

73. All surfaces shown in the figure are smooth. System is released with the spring unstretched. In equilibrium,
compression in the spring will be

a)
k
mg
2
b)
k
mg 2
c)
( )
k
g m M
2
+
d)
k
mg


74. A sphere of mass m is held between two smooth inclined walls. For sin37
0
= 3/5, the normal reaction of the
wall (2) is equal to

a) mg b) mg sin 74
0
c) mg cos 74
0
d) None of these

QUESTION BANK
17
Objective Questions wit more than one correct choice
75. A uniform rope of mass m hangs freely form a ceiling. A monkey of mass M climbs up the rope with an
acceleration a. The force exerted by the rope on the celling is
a) Ma + mg b) M(a + g) + mg
c) M(a + g) d) dependent on the position of monkey on the rope

76. The system shown in the figure is released from rest. The spring gets elongated, y

a) M > m b) M > 2m c) M > m/2
d) For any value of M (Neglect friction and masses of pulley, string and spring).

77. In the figure s > d. Masses m
1
and m
2
are connected by a light inextensible string passing over a smooth pulley.
When the system is released, then mass m
1
will acceleration towards the pulley provided.

a) 3
2
1
<
m
m
b) 3
2
1
>
m
m
c)
2
3
2
1
<
m
m
d) None of these

78. At the instant t = 0 a force F = kt (k is constant acts on a small body of mass m resting on a smooth horizontal
plane. The time when body leaves the surface, is

a)
k
mg o sin
b)
mg
k o sin
c)
o sin k
mg
d) o sin k mg

79. Certain force gives mass m
1
an acceleration a
1
and mass m
2
and acceleration a
2
. Then the acceleration the force
would give to an object of mass M will be
QUESTION BANK
18
a)
2 1
2 1
2 1
, m m M if
a a
a a
+ =
+
b)
1 2
2 1
2 1
, m m M if
a a
a a
=


c)
2 1 2 1
, m m M if a a = d)
2
1
2
1
,
m
m
M if
a
a
=

80. The acceleration of blocks of mass 5 kg and 10kg are

a) zero, if F = 100 N b) a
1
=5 m/s
2
and a
2
= 0, if F = 300N
c) a
1
= 15 m/s
2
, a
2
= 2.5m/s
2
, if F = 500N d) Acceleration of the masses are independent of F

81. Five identical cubes each of mass m are on a straight line two adjacent faces in contact on a horizontal surfaces
as shown in fig. Suppose the surface is frictionless and a constant force P is applied from left to right to the end
face of A; which of the following statement are correct?

a) The acceleration of the system is 5 P/m
b) The resultant force acting on each cube is P/S
c) The force exerted by C and D is 2P/5
d) The acceleration of the cube D is P/Sm

Match Type Questions
82. Match the acc. of block m in each ease.
List I List II
I) A) a = g +
QUESTION BANK
19
II) B) g |

III) C) g / 3 +

IV) D) a = 0

a) I-A, II-B, III-C, IV-D b) I-A, II-C, III-B, IV-A
c) I-B, II-C, III-D, IV-A d) I-D, II-B, III-A, IV-C

Comprehension Questions Passage -1:
Two pulleys are arranged as shown in fig. (pulleys are massless and frectionless). P
1
is being
pulled up by a time varying force F.
83. The velocity of 5kg block the moment 1kg is lifted up is

a) zero b) 225 m/s c) 750 m/s d) 1500 m/s

84. The velocity of 1kg block the moment 5kg is lifted up, is
a) zero b) 225 m/s c) 750 m/s d) 1500 m/s

85. The height gained by 1kg block at t = 30 sec is
QUESTION BANK
20
a) 2250 m b) m
4
15000
c) m
12
17000
d) zero
86. The height gained by 5kg block at t = 30sec is
a) 2250 m b) m
4
15000
c) m
12
17000
d) zero

87. If the string attached to surface can sustain max. force up to 35 N, then max. time the system will sustain is
a) 70 sec. b) 140 sec. c) 700 sec. d) 35 sec.


Numerical Problems
88. Fig. shows a block of mass m attached to a spring of force constant k and connected to ground by two strings
making an angle 90
0
with each other. In relaxed state natural length is l. In the situation shown in fig. find the
tension in the two strings.


89. Fig. shows a block A on a smooth surface attached with a spring of force constant k to the ceiling. In this state
spring is in natural length l. The block A is connected by a massless and friction less string to another identical
mass B hanging over a light and smooth pulley. Find the distance moved by A before it leaves contact with the
ground.


Assertion & Reason Type Questions
Directions. Q: 82-86 : The following questions consist of two statements. One labeled as Assertion and other as
Reason. Consider these statements and answer.
a) If both Assertion and Reason are true and Reason is correct explanation of Assertion
QUESTION BANK
21
b) If both Assertion and Reason are true but Reason is not correct explanation of Assertion
c) If Assertion is true but Reason is false. d) If both Assertion and Reason are false
90. Assertion: Forces of action and reaction do not cancel each other. Each produces its own effects
Reason: Forces of action and reaction are equal and opposite.
a) b) c) d)

91. Assertion: A body under the influence of concurrent forces in equilibrium either remains at rest
or moves with a constant velocity.
Reason: Concurrent forces are said to be in equilibrium when the magnitude of the resultant
force is zero.
a) b) c) d)

92. Assertion: A block of mass M is suspended by a light cord C and a stronger cord D is attached
to the lower end. A sudden jerk is given to D, then block remains at its place.
Reason : Tension in chord C is less than that in D

a) b) c) d)

93. Assertion: Two bodies of mass 50g and 20g are allowed to fall from the same height. If air resistance for each
is same, then both the bodies reach the earth simultaneously.
Reason : Acceleration of both the bodies is same.
a) b) c) d)

94. Assertion: A bird is sitting on the floor of a wire cage and the cage is in the hand of a boy. Even when the bird
starts flying in the cage, the boy does not experience any change in the weight of the cage
Reason : Bird is still in the cage because of which the boy does not experience any change.
a) b) c) d)

NEWTONS LAWS
95. In each of the following arrangements, find (a) acceleration of the system of blocks
(b) tension in each string
QUESTION BANK
22
(i) ii) iii)

iv) v) vi)

vii) viii) ix)

96. In each of the arrangements given, find the acceleration of each particle just after the force/s shown in the
figure applied. All the particles were initially at rest. You may leave the answer in terms of x and y
components. Neglect gravity if not mentioned specifically. Assume that the block does not leave the table.
a) b) c) d)

e) f) g) h)

QUESTION BANK
23
97. In each of the following arrangements, find the acceleration of each block and tension in the thread/s just after
the blocks are released from rest. Consider gravity
a) b)

98. Two small particles are restricted to move along the edges of a smooth wire cube as shown in the figure. They
are also connected by an ideal thread. If the mass of each particle is M, find the acceleration of each particle
just after the system is released from rest. One of the system is released from rest. One of the sides containing
particle is horizontal and other is vertical.


99. In each of the following cases, some particles are moving along a rope as shown in the figures. Find the tension
in all the sections of the rope in each case. Neglect gravity in parts a, e, f. All acceleration are given w.r.t the
thread.
a) b) c) d)

e) f)

100. In the following thread/pulley systems, calculate the acceleration of each mass and tension in each thread.
Assume that all the threads and pullies are ideal and that the systems are released from rest.
QUESTION BANK
24
a) b) c) d)

e) f)

g) h)

i) j)

101. In each of the following cases, find the net force applied by the threads on each of the clamps.
a) b) c)
QUESTION BANK
25



ROTATIONAL MOTION (Single option correct)
102. Find moment of inertial of a semicircular disc of mass m and radius R about an axis passing through point P
about an axis passing through point P (see Fig.) of semi circle and perpendicular to plane of the disc

a)
4
2
MR
b)
2
2
4
t
MR
c)
|
.
|

\
|

t
4
2
3
2
MR d)
|
.
|

\
|

t
4
3
2
2
MR

103. A force of magnitude 20 unit acts along the vector 2i 3j + 4k at point i + j + 2k. Find magnitude of torque
about y-axis.
a)
29
5 60
b)
29
3 100
c)
20
5 29
d)
20
3 29


104. A triangle ABC where AB = 3, BC = 4, CA = 5 units and G is the centroid to triangle. The minimum and
maximum moment of inertia will be about line
a) AG, CG b) CG, BG c) BG, AG d) BG, CG

105. For a body making rotation according to
2
15 Bt At + = e . If the angular momentum vector reverts its
direction twice at an interval of sec 2 = At , and the torque is zero at t = 4sec, then the values of A and B are
a) 1, 8 b) 8, 1 c) 4, 1 d) 1, 4

106. A rod of uniform mass of length L can freely rotate in a vertical plane about an axis passing through O. The
angular velocity of the rod when it falls from position P to P through an angle o is

a) o sin
5
6
L
g
b)
2
sin
6 o
L
g
c)
2
cos
6 o
L
g
d) o sin
6
L
g

QUESTION BANK
26

107. A light thin hoop of radius R is attached with six point mass equal to m each according to fig., moving without
slipping on the track. Find the angular momentum of mass B about the point of contact P of the hoop when the
mass A at the point P, take speed of D equal to 2v.

a)
2
3 mvr
b)
2
mvr
c) mvr d)
3
2
mvr


108. Two point masses of mass m and speed v stick to an equilateral triangle made by three massless rods at the end
B and C. The system is placed at smooth plane and masses stick simultaneously moving normal to sides. The
angular velocity after the collision will be

a)
2
3v
b)
3
2v
c)
3
v
d) v 2

109. A light cylinder placed on a surface with coefficient friction 5 . 0 = is attached with a thin rod of mass m is
released from rest as shown in Fig. Find the acceleration of centre of rod after the release

a)
2
g
b) g c) 0 d) g 2

110. A sphere of mass M and radius r (see Fig.) slips on a rough horizontal plane. At some instant it has
translational velocity v
0
and rotational velocity about the centre v
0
/2r. The translational velocity after the
sphere starts pure rolling, is
QUESTION BANK
27

a) 6v
0
/ 7 in forwards direction b) 6v
0
/ 7 in backwards direction
c) 7v
0
/ 7 in forwards direction d) 7v
0
/ 6 in backwards direction

111. A particle of mass m = 5kg is moving with a uniform speed of s m v / 2 3 = in xy plane along the line y = (x
+ 4). The magnitude of the angular momentum (in kg. m
2
/sec) about origin is:
a) zero b) 60 c) 7.5 d) 2 40

112. A ring of mass 0.3kg and radius 0.1m and a solid cylinder of mass 0.4kg and of the same radius are released
simultaneously on a flat horizontal surface such that they begin to roll with the same KE as soon as released
towards a wall which is at the distance from the ring and the cylinder. Then
a) the cylinder will reach the wall first b) the ring will reach the will first
c) Both will reach the wall simultaneously d) None of these

113. A thin uniform L shape rod of mass 4kg is hinged as shown. When the thread is cut find the initial acceleration
of point A.

a)
2 2
3g
b)
5
2 3g
c)
2
3g
d)
3
2g


More than one options may be correct
114. A thin rod of length 6m & mass 6m is given a sharp impulse of 120 N-sec at point 2m from an end according
to fig. The rod is placed on a smooth horizontal plane. Just during impulse rod breaks up in two parts at the
point of impulse. Assuming impulse to be divided equally in both parts the relative angular velocity of parts
will be
QUESTION BANK
28

a)
sec
30
rad
b)
sec
150
rad
c)
sec
120
rad
d) None of these

115. Two similar particles A and B rotate in xy plane of radius r
A
and yz plane of radius r
B
around vertical axis
passing through the centre of their orbits. The uniform angular velocity of A is
A
e and that of B is
B
e , then
relative angular acceleration and angular velocity are

a)
A B
e e b)
2 2
2
1
,
4
B A
A B
e e
e e
+ c)
2 2
B A
e e + d)
2 2
2
B A B A
e e e e +

116. Two persons of equal mass m are sliding freely on ice, each with a speed v on parallel straight path in opposite
directions. The paths are separated by a perpendicular distance d. the first person carries a pole of length l held
firmly at one end. The second person grips the other end of the pole just as they are passing each other. Which
of the following statements are true for the subsequent motion in a circular path?
a) Each person moves in a circular path of radius l
2
1
with constant speed v.
b) Each persons moves in a circular path of radius
2
l
about a centre which itself moves with
speed v.
c) The tension in the pole remains constant
d) They come to rest when the pole has rotated through
2
t
to lie along the direction of original
motion.

117. The torque t on a body about a given point is found to be equal to A x L where A is a constant vector and L is
angular momentum of the body about that point from this. It follows that
a)
t d
l d
is perpendicular to L at all instant of time.
QUESTION BANK
29
b) The component of L in the direction of A does not change with time.
c) The magnitude of L does not change with time
d) L does not change with time

118. A ball rolls down an inclined plane and acquires a velocity v
r
, when it reaches the bottom of the plane. If the
same ball slides without friction on other plane and acquires rolling from the same height down an equally
inclined smooth plane and acquires a velocity v
s
, which of the following statements is/are correct?
a) v
r
< v
s
because a work is done by the rolling ball against the frictional force.
b) v
r
> v
s
because the angular velocity acquired makes the rolling ball to travel faster.
c) v
r
= v
s
because kinetic energy of two balls is same at bottom of plane.
d) v
r
< v
s
, because the rolling ball acquires rotational as well as translational kinetic energy.

119. Two similar thin rods of mass m each making an angle of 120
0
are hinged at point P (see Fig.) when a F = 6mg
is applied to hinge at angle of 120
0
form both rods. (Neglect the effect of gravity). For t = 0sec. select the
correct options.

a) Linear acceleration of centre of rods will be 3g each.
b) Acceleration of point A will be 3g
c) Acceleration of point A will be more than 3g
d) Linear acceleration of centre of rods will be less than 3g

120. A uniform disc of radius 1m is first allowed to spin about its axis with angular velocity of 4 rad/s and then
carefully placed with its flat face on a horizontal surface. The coefficient of friction is 3 / 1 = . Take g =
10m/s
2
. Then
a) the angular retardation produced will be (40/9) rad/s
2

b) the angular retardation produced will be (10/3) rad/s
2

c) the disc will stop rotating after t = 0.9 sec
d) the disc will stop rotating after t = 2 sec

121. A ring rolls without slipping on a horizontal surface. At any instant, its position is as shown in the fig. Then
QUESTION BANK
30

a) section ABC has greater kinetic energy than section ADC.
b) section BC has greater energy than section CD
c) section BC has the same kinetic energy as section DA.
d) the section AB, BC, CD and DA have the same kinetic energy

Comprehension -A
Force of friction plays a role of turning force in motion of a rolling rigid body. The force of friction always
tries to oppose the relative motion between the two surfaces in contact. As a force it can produce linear
acceleration as well
122. A solid sphere rotating with an angular speed e is put on a long plank of mass equal to sphere. The friction
exists between plank and sphere while plank rests on a plane frictionless surface, then
a) speed of centre of sphere will increase first then decrease.
b) speed of centre of sphere will increase then become constant.
c) the plank will have same acceleration as sphere will have.
d) speed of centre of mass of the system will be half of that of speed of centre of sphere.




123. Choose the correct options(s)
a) Total kinetic energy will increase
b) Total kinetic energy will decrease
c) Translational kinetic energy of the system will increase
d) Total energy will be remain same

124. Choose the correct options (s)
a) The angular momentum of sphere and plank will not conserve.
b) The angular momentum will conserve only about the centre of sphere.
c) The angular momentum will conserve about all points.
d) Angular momentum of the system will conserve only about the point of contact where force
of friction acts
QUESTION BANK
31
From stational K.E. of the plank will be equal to
a) b) c) d)

Comprehension -B
A solid cylinder of radius R and mass M rolls over an inclined plane with inclination
0
45 = u with its axis
horizontal. Let the friction coefficient between the plane and cylinder be and length of plane to travel be L
= 20m. Then if cylinder starts with rest, what will be the parameters of motion? (take g = 10m/sec
2
)
125. Choose the correct option(s)
a) If were greater than 1/3 the motion would be pure rolling
b) If is less than 1/3 there will be loss of kinetic energy during motion
c) If 0 > or 3 / 1 > , there will be no loss of mechanical energy
d) There will always be a loss of mechanical energy whatever be the value of

126. If =1/6, relative velocity of point of contact at time t, is
a)
3
2 20 t
b) t
2 2
10
c) t
3
30
d)
3
2 5 t


127. Mechanical energy loss will be maximum for the value of equals to
a)
3
1
b)
4
1
c)
6
1
d)
12
1


128. Loss of mechanical energy in above case in S.I. units is
a) 2 10 m b) 2 100 m c)
2
100 m
d)
2
10 m


Comprehension -C
Turning effect of a force is given by torque, where F r = t . A force may produce turning as well as linear
effect. Consider a solid sphere of mass m and radius r in space where there is no gravity or any other force, a
force F is applied tangentially to the sphere by some mean the direction of force does not change by time.

QUESTION BANK
32
129. The solid sphere will move in a
a) straight line b) circular path c) toroide path d) parabolic path

130. The solid sphere will have
a) linear accelerate only b) angular acceleration only c) will have both d) None of these

131. Choose the correct option
a) Linear acceleration
m
F
= b) Angular acceleration
mr
F
2
5
=
c) Both of these d) No angular acceleration

132. The sphere will have kinetic energy in time
a)
m
t F
2
2 2
b)
m
t F
2 2
8
7
c)
m
t F
2 2
4
7
d)
m
t F
4
2 2


Subjective Questions
133. A thin uniform rod AB of mass m = 1.0kg moves translationally with acceleration
2
/ 0 . 2 s m = o due to two
antiparallel force F
1
and F
2
(fig.). The distance between the points at which these forces are applied is equal to
a = 20cm. Besides, it is known that F
2
= 5.0 N. Find the length of the rod.


134. A sphere, a disc and a ring of the same mass and radius are rolled from rest from the top of a frictionless
inclined plane simultaneously. In which order do they reach the bottom of the incline?

135. A uniform ladder of mass 20kg and length 4m rests against a smooth wall where friction coefficient with
ground be 5 . 0 = . If g = 10m/sec
2
, what maximum angle ladder could make with vertical measure angle in
degrees?

QUESTION BANK
33
136. A uniform rod of mass 5.0kg and length l = 90cm, rests on a smooth horizontal surface, one of the end of the
rod is struck with the impulse J = 3.0 N-s in horizontal direction perpendicular to the rod. As a result, the rod
obtains the momentum p = 3.0 N-s. Find the K.E. with which rod will move.

Matching Type Questions
137. List I List II
For a body rolling down on a inclined Here t
1
< t
2
< t
3
< t
4
mass and radius are
plane with out slipping, the time taken same for all bodies
to reach bottom will be
I) Ring A) t
3

II) Solid sphere B) t
2

III) Solid cylinder C) t
4

IV) Hollow sphere D) t
1

a) I-D, II-C, III-B, IV-D b) I-C, II-D, III-B, IV-A
c) I-D, II-B, III-D, IV-C d) I-D, II-B, III-A, IV-C


138. List I List II
I) Torque A) r x p
II) Angular Momentum B)
2
e L

III) Power C) r x F
IV) Energy D) t e
a) I-C, II-A, III-B, IV-D b) I-A, II-C, III-D, IV-B
c) I-C, II-A, III-D, IV-B d) I-A, II-C, III-B, IV-D

139. List I List II
I) A) MI will be mr
2
3

II) B) ( )
2
2
2
1
4
R R
M
+
QUESTION BANK
34
III) C) ( )
2
2
2
1
5
4
R R
M
+
IV) D) ( )
2
2
2
1
2
R R
M
+
a) I-B, II-D, III-C, IV-A b) I-B, II-C, III-D, IV-A
c) I-D, II-A, III-B, IV-C d) I-C, II-B, III-D, IV-A

140. List I List II

I) Torque angular acceleration, angular A) Tensor
momentum, angular velocity
II) Momentum of inertia B) Radial vectors
III) Angular displacement of finite value,
rotational kinetic energy C) Axial vector
IV) Centripetal acceleration, centripetal force D) Not vectors
a) I-C, II-A, III-D, IV-B b) I-B, II-D, III-A, IV-C
c) I-A, II-C, III-B, IV-D d) I-B, II-A, III-D, IV-C



141. A block of mass 0.1kg is held against a wall by applying a horizontal force of 5N on the block. If
the coefficient of friction between the block and wall is 0.5, the magnitude of frictional force
acting on the block is
a) 2.5 N b) 0.98 N c) 4.9 N d) 0.49 N

142. A block of mass 2kg rests on a plane inclined at an angle of 30
0
with the horizontal. The
coefficient of friction between the block and surface is 0.7. What is the frictional force acting on
the block?
a) 11.9 N b) b) 9.82 N c) 15.75 N d) 5.8 N

QUESTION BANK
35
143. A 5kg stationary bomb is exploded in three parts having mass 1 : 1 : 3 respectively. Parts
having same mass move in perpendicular direction with velocity 39 m/s, then the velocity of
bigger part is
a) s m/
2
13
b) s m/ 2 10 c) s m/
2
10
d) s m/ 2 13

144. A ship of mass 3 x 10
7
kg initially at rest is pulled by a force of 5 x 10
4
N through a distance of
3m. Assuming that the resistance due to water is negligible, what will be the speed of ship?
a) 0.1 m/s b) 1.5 m/s c) 5 m/s d) 0.2 m/s

145. Two masses of 10kg and 20kg respectively are tied together by a massless spring. A force of 200N is applied
on a 20kg mass. At the instant shown, the acceleration of 10kg mass is 12m/s
2
, the acceleration of 20kg mass is

a) 0 m/s b) 10 m/s
2
c) 4 m/s
2
d) 12 m/s
2


146. A board of mass 5kg is suspended from a horizontal beam by two supporting wires, each at an angle of 45
0
to
the vertical. The tension of each wire is (g = 10ms
2
)

a) 50 N b) 35 N c) 12.5 N d) 20 N

147. A given object takes n times more time to slide down a 45
0
rough inclined plane as it takes to slide down a
perfectly smooth 45
0
incline. The coefficient of kinetic friction between the object and incline is
a)
2
1
1
n
b)
2
1
1
n
c)
2
1
1
n
d)
2
1
1
n


148. In a rocket, fuel burns at the rate of 1kg/s. This fuel is ejected form the rocket with a velocity of 60km/s. This
exerts a force on the rocket equal to
a) 60 N b) 600 N c) 6000 N d) 60000 N
QUESTION BANK
36

149. A block B of mass M = 15kg hangs by a cord from a knot K of mass m
k
which hangs from a ceiling by means
of two other cords. The cords have negligible mass, and the magnitude othe gravitational force of the knot is
negligible compared to the gravitational force. The tensions in the three cords are

a) N T N T N T 104 , 134 , 147
3 2 1
= = = b) N T N T N T 147 , 134 , 104
3 2 1
= = =
c) N T N T N T 147 , 104 , 134
3 2 1
= = = d) N T N T T 147 , 134 , , 0
3 2 1
= = =

150. There are two force on the 2.0kg box. Only one force is shown. The figure also shows the acceleration of the
box. The magnitude of second force along with direction is

a) 38 N, 213
0
b) 20 N, 213
0
c) 42 N, 180
0
d) 58 N, 180
0


151. An 11kg salami is supported by accord that runs to a spring scale, which is supported by another cord form the
ceiling (Fig.1). The spring scale gives reading in weight units. Now the salami is supported by a cord that runs
around a pulley and to a scale (as shown in Fig.). In third case the wall has been replaced by a second 11kg
salami on left, and assembly is stationary. The reading of the scale in the three cases is [ g = 10m/s
2
]

a) 11 kg, 22kg, 44kg b) 110kg, 220kg, 440kg c) 11kg, 44kg, 55kg d) Same in all three cases
QUESTION BANK
37

152. When a nucleus captures a stray neutron, it must bring the neutron to a stop within the diameter of the nucleus
by means of the strong force. That force which glues the nucleus together is approximately zero outside the
nucleus. Suppose that a stray nucleus with an initial speed of 1.4 x 10
7
m/s is just barely captured by a nucleus
with diameter d = 1.0 x 10
-14
m. Assuming that the strong force on the neutron is constant, the magnitude of this
force is
[Given neutrons mass is 1.67 x 10
-27
kg]
a) 4 N b) 16 N c) 8 N d) 32 N

153. A 40kg girl and an 8.4kg sled are on the frictionaless ice of a frozen lake, 15m apart but connected by a rope of
negligible mass. The girl exerts a horizontal 5.2N force on the rope. Then how far from the girls initial
position do they meet?
a) 1.3 m b) 6.2 m c) 2.6 m d) Data Insufficient

154. A sphere of mass 3.0 x 10
-4
kg is suspended form a cord. A steady horizontal breeze pushes the sphere so that
the cord makes a constant angle of 37
0
with the vertical. The tension in the cord is given by
a) 3.7 x 10
-3
N b) 2.2 x 10
-3
N c) 6.8 x 10
-3
N d) 8.5 x 10
-3
N

155. Two blocks are in contact on a frictionaless table as shown. A horizontal force (F = 3.2N) is applied to the
larger block. A force of same magnitude is applied to the block of smaller mass but in opposite direction. The
force between the blocks in first case is
2 1
f and f in second case. Also given m
1
= 2.3kg, m
2
= 1.2kg. Then,
which one of the following holds true?

a)
2 1
f f = b)
2 1
f f > c)
2 1
f f < d)
2
2
1
f
f =

156. A 5.00kg block is pulled along a horizontal frictionless floor by a cord that exerts a force of magnitude F =
12.0 N at an angle
0
25 = u above the horizontal. The magnitude of blocks acceleration just before it is lifted
(completely) off the floor is

QUESTION BANK
38
a) 2.18 m/s
2
b) 21 m/s
2
c) 5.18 m/s
2
d) 11 m/s
2


157. A chain consisting of five links, each of m = 0.100kg is lifted vertically with a constant acceleration of 2.50
m/s
2
. The magnitude of the force on link 3 form link 4 is

a) 2.46 N b) 0.25 N c) 3.69 N d) 4.92 N

158. If a cars wheels are locked (kept from rolling) during emergency braking, the car slides along the road.
Ripped-off bits of tire and small melted sections of road form the skid marks that reveal that cold welding
occurred during the slide. The skid mark was 290m long. Assuming that cars acceleration is constant during
the braking and 60 . 0 = , how fast was the car going when the wheels become locked?

a) 90 km/hr b) 210 km/hr c) 80 km/hr d) 120 km/hr

159. A man pulls a loaded sled of mass m = 75kg along a horizontal surface at constant velocity. The coefficient of
kinetic friction
k
between the runners and the snow is 0.10 and angle is 42
0
. What is the magnitude of the
force
a
T on the sled from the rope?

a) 91 N b) 41 N c) 22 N d) 0 N

160. A coin of mass m is at rest on a book that has been tilted at an angle u with the horizontal. When u is
increased to 13
0
, the coin is on the verge of sliding down the book, which means that even a slight increase
beyond 13
0
producing sliding. The coefficient of static friction between the coin and book is
QUESTION BANK
39

a) 0.5 b) 0.33 c) 0.19 d) 0.23

161. A 1.34kg sphere is connected by means of two massless strings, to a vertical, rotating rod. The strings are tied
to the rod and are taut. The tension in the upper string is 35 N. The tension in the lower string is
a) 8.74 N b) 6.45 N c) 9.98 N d) 7.48 N

162. A 1000 kg boat is traveling at 90 km/hr when its engine is shut off. The magnitude of the frictional force
a
k
f

between boat and water is proportional to the speed of the boat.
( ) v f
k
70 =
where v is in meters per second
and
k
f
in Newtons. The time required by the boat to slow to 45 km/hr is
a) 1 s b) 5 s c) 12.8 s d) 9.9 s

163. The mass of an elevator (lift) is 500 kg. The tension in the cable elevator when the elevator is ascending with
an acceleration of 2ms
-2
is
a) 5900 N b) 4900 N c) 3900 N d) 2900 N

164. The masses M
1
, M
2
, M
3
of three bodies are 5, 2 and 3 kg. The values of tensions T
1
, T
2
and T
3
will be how
much, when the whole system is going upward with an acceleration of 2ms
-2
?

a) T
1
= 29.4 N, T
2
= 98 N, T
3
= 98 N b) T
1
= 98 N, T
2
= 49 N, T
3
= 29.4 N
c) T
1
= 35.4 N, T
2
= 118 N, T
3
= 59 N d) T
1
= 118.4 N, T
2
= 59 N, T
3
= 35.4 N

QUESTION BANK
40
165. A small cart with a sphere suspended over it by a string approaches an inclined plane at a speed v. In which
direction, with respect to the vertical will the string supporting the sphere be deflected when the cart begins to
climb the cart begins to climb the inclined plane?

a) will remain vertical. b) will be perpendicular to the inclined plane.
c) will be horizontal. d) none of these

166. A block A of mass 7kg placed on a frictionless table. A thread tied to it passes over a frictionless pulley and
carries a body B of mass 3kg at the other end. The acceleration of the system is (g = 10 ms
2
)

a) 30 m/s
2
b) 3 m/s
2
3) 10 m/s
2
4) 1 m/s
2


167. A particle starts moving in a straight line with constant acceleration a. At a time t
1
second after the beginning
of motion, the acceleration changes sign, remaining the same in magnitude. Determine the time from the
begining of motion, till it returns to the starting point.
a)
( ) 2 2
1
t
b)
2
1
t
c)
2
1
t
d)
( ) 2 2
1
+ t


168. A person walks up a stationary escalator in 90 seconds. If the escalator moves with person, first standing on it,
it will take 1 minute to reach the top from ground. How much time would it take him to walk up the moving
escalator?
a) 3 s b) 12 s c) 36 s d) 48 s

169. A block slides down a rough inclined plane of slope angle
|
with constant velocity. It is then projected up the
same plane with an initial speed v
0
. How far up the plane will it move before coming to rest?
QUESTION BANK
41

a)
| sin
2
0
g
v
b)
| sin 4
2
0
g
v
c)
2
sin| g
d)
| sin
2
2
0
g
v


170. A block of mass m slides in an inclined right-angled trough as shown. If the coefficient of kinetic friction
between block and material composing the trough is
k

, find the acceleration of block.



a)
( ) u u cos 2 sin
k
g
b)
( ) u u cos 2 sin
k
g

c)
( ) u u sin cos
k
g
d)
( ) u u cos sin 2
k
g


171. An aeroplane of mass 10,000 kg requires a speed of 20ms
-1
, for take off the run on the ground being 100m. The
coefficient of kinetic friction between the wheels of the plane and ground is 0.3. Assume that plane accelerates
uniformly during take off. The minimum force required by the engine to take off
a) 2 x 10
4
N b) 5 x 10
4
N c) 3 x 10
4
N d) 4 x 10
4
N

172. A caterpillar crawls up a fixed hemispherical bowl of radius R. If the coefficient of friction is
3
1
. Find the
height as percentage of radius upto which caterpillar can crawl.
a) 3% of R b) 5% of R c) 7% of R d) 9% of R

ROTATIONAL DYNAMICS-II (Practice Sheet-3)
173. In each of the following mass/pulley systems, mass of each pulley is m and radius is r. All threads are massless
and there is sufficient friction between threads and pullies to prevent slipping between them. There is no
friction between any other surface. Treat each pulley as a uniform disk. In each case, find the acceleration of
each mass after the system is released form rest.
QUESTION BANK
42
a) b) c)

d) e)



174. Figure shows two blocks of masses m and M connected by a string passing over a pulley. The horizontal table
over which the mass m slides is smooth. The pulley has a radius r and moment of inertia I about its axis and it
can freely rotate about this axis. Find the acceleration of the mass M assuming that the string does not slip on
the pulley


175. A string is wrapped on a wheel of moment of inertia 0.20kg-m
2
and radius 10cm and goes through a light
pulley to support a block of mass 2.0kg as shown in the figure. Find the acceleration of the block.


176. The pulley shown in the figure has a radius 10cm and moment of inertia 0.5kg-m
2
about its axis. Assuming the
inclined planes to be frictionless and fixed, calculate the acceleration of the 4.0kg block.
QUESTION BANK
43


177. Solve the previous problem if the friction coefficient between the 2.0kg block and the plane below it is 0.5 and
the plane below the 4.0kg block is frictionless.

178. A light rod of length 1m is pivoted at its centre and two masses of 5kg and 2kg are hung from the ends as
shown in the figure. Find the initial angular acceleration of the rod assuming that it was horizontal in the
beginning.


179. Suppose the rod in the previous problem has a mass of 1kg distributed uniformly over its length. (a) Find the
initial angular acceleration of the rod. (b) Find the tension in the supports to the blocks of mass 2kg and 5kg.

180. In each of the following cases, some block/s is/are attached to a step pulley. In each case, taking moment of
inertia of the step pulley to be I, find its angular acceleration. All pullies except step pullies are massless and
frictionless.
a) b) c)

181. Two objects are attached to ropes attached to wheels on a common axle as shown in the figure. The total
moment of inertia of the two wheels is 40kg-m
2
. The radii are R
1
= 1.2m and R
2
= 0.4m.
QUESTION BANK
44

a) If m
1
= 24 kg, find m
2
such that the system is in equilibrium
b) If 12 kg is gently added to the top of m
1
, find the angular acceleration of the wheels and the tension in the
ropes.

ROTATIONAL DYNAMICS-II (Practice Sheet-4)
182. In each of the following cases, a body is rotating about a given axis with uniform angular speed e . Find the
reaction force applied by axis of rotation on the body at the instant line joining points A and B is parallel to x-
axis. Neglect gravity and friction.
a) b)

c) d)


183. Each of the given arrangements shows a body that is free to rotate about a horizontal axis. The body is initially
held at rest and released from the position shown in the figure. In each case, just after the release, find
i) angular acceleration of the body.
ii) acceleration of centre of mass of the body
QUESTION BANK
45
iii) force being applied on the body by the axis of rotation
a) b) c) d)

184. A rod AB of length L and mass m is lying horizontally on a horizontal frictionless table. The rod is pivoted
about the end A. Suppose that a horizontal force of magnitude F is applied on end B, such that is always
perpendicular to the length of the rod. Just after the force is applied, find:
a) angular acceleration of the rod. b) angular speed of the rod
c) acceleration of centre of mass of the rod. d) force being applied by the rod on the pivot
e) force of interaction between a small particle of mass dm at the midpoint of the rod and rest of
the rod. f) force applied by one half of the rod on the other.

185. Three particles A, B and C, each of mass m, are connected to each other by three massless rigid rods to from a
rigid, equilateral triangular body of side l. This body is placed on a horizontal frictionless table (x-y plane) and
is hinged to it at the point A so that it can move without friction about the vertical axis through A (see fig.).
The body is set into rotational motion on the table about A with a constant angular velocity e .

a) Find the magnitude of the horizontal force exerted by the hinge on the body
b) At time T; when the side BC is parallel to the x-axis, a force F is applied on B along BC
(as shown). Obtain the x-component and the y-component of the force exerted by the hinge
on the body, immediately after time T.

186. A uniform rod AB of mass m = 2kg and length l = 100cm is placed on a sharp support O such that AO = a =
40cm and OB = b = 60cm. A spring of force constant K = 600 Nm
-1
is attached to end B as shown in the
figure. To keep the rod horizontal, its end A is tied with a thread such that the spring is elongated by y = 1cm.
Calculate reaction of support O on the rod when the thread is burnt (g = 10 ms
-2
).
QUESTION BANK
46


187. Frame ABC shown in the figure is rotating with constant angular velocity e in a gravity freee region. At the
instant shown in the figure, find the force applied by part AB of frame on part BC. Express your answer in
terms of
^
i and
^
j
.



ROTATIONAL DYNAMICS I (Practice Sheet -1)
188. In each of the following cases, calculate the magnitude of net torque acting on the body about the given axis of
rotation. Symbol (-) denotes that the axis of rotation is perpendicular to the plane of paper and symbol
denotes that the axis of rotation is in the plane of paper.
a) b) c) d)

e) f) g) f)

QUESTION BANK
47
i) j) k)

k) m) n)


o)

189. Determine the magnitude of net torque acting on the body shown in each case about the given axis of rotation.
a) b) c)

QUESTION BANK
48
d) e) f)

g) h) i)

190. Determine the moment of following forces about the specified points. Express your answer in unit vectors
^
i ,
^
j
,
^
k . Also specify the axis of rotation in each case.
a)
^ ^
2 j i F +

acting at (4, 6) about (0, 0). b)


^ ^
2k i F +

acting at (4, 6) about (0, 0).



191. Consider a force
|
.
|

\
|
+
^ ^
6 4 j i
acting on a large sphere centred at origin. Force is acting at the point (0, 0, 2).
If the sphere can rotate about origin, find the equation of axis of rotation.

ROTATIONAL DYNAMICS I (Practice Sheet -1)
192. Consider a light rod with two heavy mass particles at its ends. Let AB be a line perpendicular to the rod as
shown in the figure. What is the moment of inertia of the system about AB?


193. Three particles, each of mass 200g, are kept at the corners of an equilateral triangle of side 10cm. Find the
moment of inertia of the system about an axis
a) joining two of the particles; and
b) passing through one of the particles and perpendicular to the plane of the particles.

QUESTION BANK
49
194. Particles of masses 1g, 2g, 3g,100g are kept at the marks 1cm, 2cm, 3cm,.100cm respectively on a metre
scale. Find the moment of inertia of the system of particles about a perpendicular bisector of the metre scale.

195. The moment of inertia of a uniform rod of mass 0.50kg and length 1m is 0.10kg-m
2
about a line perpendicular
to the rod. Find the distance of the from the middle point of the rod.

196. Two uniform identical rods each of mass m and length l are joined to from a cross as shown in the figure. Find
the moment of inertia of the cross about a bisector as shown by the dotted line dotted in the figure.


197. Three rods each of mass m and length l are joined together to from an equilateral triangle as shown in the
figure. Find the moment of inertia of the system about an axis passing through its centre of mass and
perpendicular to the plane of the triangle.


198. A rod of length l is pivoted about an end. Find the moment of inertia of the rod about this axis if the linear
mass density of rod varies as
m kg b ax /
2
+ =
.


199. A thin wire is bent to from a circular ring of radius R. The linear density of the wire varies as
u

k
e
0
=
,
t u 2 0 < s ,
0

and k are constants and u is the an angle shown in the figure. Find the moment of inertia
of the ring about an axis passing through is centre and perpendicular to is plane.

QUESTION BANK
50

200. A thin uniform wire is bent to from a spiral whose equation is given by:
t u
u
2 0 ,
0
< s =
k
e r r
. Find
the moment of inertia of the spiral piece of wire about z-axis (see figure).


201. Find the moment of inertia of the rod AB about an axis y as shown in the figure. Mass of the rod is m and
length is l.


202. A thin steel wire is bent into the shape shown. Denoting the mass per unit length of the wire by m, determine
by direct integration the moment of inertia of the wire with respect to each of the coordinate axes. (symmetry
in x, y).


203. A flat rectangular plate has variable superficial density given by
x
0
o o =
, where
0
o
is a constant and x is
the distance measured along x-axis. (see figure). Find the moment of inertia of the plate about (i) y-axis
(ii) x-axis (iii) z-axis

QUESTION BANK
51

204. Determine the moment of inertia of an ellipse of semi axes a and b, about (i) the major axis, and (ii) the minor
axis. Use the perpendicular axis theorem to determine the moment of inertia of the ellipse about an axis
perpendicular to the plane of the ellipse through the centre.


205. Determine the moment of inertia of a laminar circular sector of radius R and mass M, making an angle of 60
0

at the centre, about an axis passing through the centre of the outermost rim x and perpendicular to the plane of
the sector.


206. The mass of circular disc in m. A circular hole is now cut into it as shown in the figure. Find the moment of
inertia of the remaining part about an axis passing through point A perpendicular to the discs plane.


207. The surface density (mass/area) of a circular disc of radius a depends on the distance from the centre as
( ) Br A r + =
. Find its moment of inertia about the line perpendicular to the plane of the disc through its
centre.

208. A flat semicircular disc of radius R has variable surface density given by
y
0
o o =
where
0
o
is a constant
and y is the distance measured along y-axis. Find the moment of inertia of the plate about:
i) x axis ii) y-axis iii) z-axis

QUESTION BANK
52
209. A flat circular disc of radius R has variable density given by
u o o
2
0
sin =
where
0
o
is a constant and
u is the angle made with x-axis. Find the moment of inertia of the disc about an axis passing through its
centre, perpendicular to its plane.

210. The solid cylinder has an outer radius R, height h, and is made form a material having a density that varies
from its centre as
2
ar k + =
, where k and a are constants. Determine the mass of the cylinder and its
moment of inertia about the z-axis.


211. Find the moment of inertia of a truncated cone about its axis, the radii of its ends being a and b.

212. The paraboloid is formed by revolving the shaded area around the x-axis. Determine the radius of gyration k
x
.
The material has a constant density

.


213. The area shown is revolved about the x-axis to from a homogeneous solid of revolution of mass m. Using
direct integration, express the moment of inertia of the solid with respect to the x-axis in terms of m and h.


214. Find the moment of inertia of a hollow sphere about a diameter, its external and internal radii being a and b.

215. Determine the moment of inertia of the homogeneous triangular prism with respect to the y-axis. Express the
result in terms of the mass m of the prism.
QUESTION BANK
53

Friction in Rotational Mechanics
216. Find out the minimum friction coefficient so that the cylinder does not slip on inclined plane

217. A ladder is resting over the rough wall and rough surface. Find out the angle u for equilibrium.


218. Find out min. friction coefficient to topple this hexagon on rough surface


219. Find out force F required to maintain the plank over two rollers.


QUESTION BANK
54
220. Find out the max. angle u for equilibrium of hemisphere resting at the corner of a room, take friction
coefficient for each surface.

221. A disc of mass M and radius R is in pure rolling by the application of force F at height h. Decide how the
friction force changes its direction w.r.t. height of force application.


222. Discuss the minimum friction coefficient for objects (i) Ring (ii) Disc (ii) Solid sphere (iv) Spherical shell
rolling on inclined plane about bottom most point


223. Find out the minimum friction coefficient to prevent slipping


224. Find out the maximum value of T for pure rolling of spool. Spool may be considered as disc of radius R and
mass M.


QUESTION BANK
55

225. Find out the acceleration of plank and cylinder and their friction forces for pure rolling


226. A solid sphere and a thin spherical hoop of equal mass m and radius R are harnessed together by rigging and
free to roll without slipping down the inclined plane as shown. Neglecting the mass of rigging, determine the
force in rigging. Assume frictionless bearing.


227. A rope is wrapped over the rough pulley ad tries to rotate pulley. Friction coefficient between them is and
the angle made by rope at centre is o . Find out how T
1
is related to T
2
.


228. Consider a solid sphere and cube are resting on sufficient rough inclined surface. Discuss the normal contact
force between them



O Level (Multiple choice Questions)
229. A round disc of moment of inertia I
2
about its axis perpendicular to its plane and passing through its centre is
placed over another disc of moment of inertia I
1
rotating with an angular velocity e about the same axis. The
final angular velocity of the combination of discs id
QUESTION BANK
56
a)
( )
1
2 1
I
I I e +
b)
2 1
1
I I
I
+
e
c) e d)
2 1
1
I I
I
+
e


230. A solid sphere, a hollow sphere and a ring are released form top of an inclined plane (frictionless) so that they
slide down the plane. Then maximum acceleration down the plane is for (no rolling)
a) Solid sphere. b) Hollow sphere c) Ring d) All same

231. A particle of mass m moves along line PG with velocity v as shown. What is the angular momentum of the
particle about P?

a) mvL b) mvl c) mvr d) Zero

232. A circular disc X of radius R is made form an iron plate of thickness t and another disc Y of radius 4R is made
form an iron plate of thickness t/4. Then the relation between the M.I. I
x
and I
y
is
a) I
y
= 32 I
x
b) I
y
= 16 I
x
c) I
y
= I
x
d) I
y
= 64 I
x


233. A ball rolls without slipping. The radius of gyration of the ball about an axis passing through its centre of mass
is k. If the radius of ball be R, then the fraction of total energy associated with its rotational energy will be
a)
2
2 2
R
R k +
b)
2
2
R
k
c)
2 2
2
R k
k
+
d)
2 2
2
R k
R
+


234. A circular disc of radius R and thickness R/6 has moment of inertia. I. about an axis passing through its centre
and perpendicular to its plane. It is melted and recast into a solid sphere. The M.I. of the sphere about its
diameter as axis of rotation is
a) I b) 2I / 8 c) I / 5 d) I / 10

235. A heavy disc is thrown on a horizontal surface in such a way that it slides with speed v
0
initially without
rolling. It will start rolling without slipping when its speed reduces to
a) v
0
/2 b) 2v
0
/5 c) 3 v
0
/5 d) 5 v
0
/7

QUESTION BANK
57
236. One quarter sector is cut from a uniform circular disc of radius R. This sector has mass M. It is made to rotate
about a line perpendicular to its plane and passing through its centre of the original disc. Its moment of inertia
about the axis of rotation is

a)
2
2
1
MR b)
2
4
1
MR c)
2
8
1
MR d)
2
2MR

237. A disc of mass M and radius R is rolling with angular speed e on a horizontal plane as shown. The magnitude
of angular momentum of disc about the origin O is

a) e
2
2
1
MR b) e
2
MR c) e
2
2
3
MR d) e
2
2MR

238. A cubical block of side a is moving with velocity v on the horizontal smooth plane as shown in fig. It hits a
ridge at point O. The angular speed of the block after it hits O is

a)
a
v
4
3
b)
a
v
2
3
c)
a
v
2
3
d) Zero

239. A particle of mass m is released form rest at point A falling parallel to vertical y-axis. Its angular momentum
after falling through distance h about O is

QUESTION BANK
58
a) gh ma 2 b) gh
a
m
2 c)
gh
ma
d)
ag
mh

240. A cylinder of radius R is spinned and then placed on an inclined having friction coefficient u tan = . The
cylinder continues to spin without falling for time

a)
u
e
sin 3
0
g
R
b)
u
e
sin 2
0
g
R
c)
u
e
sin
0
g
R
d)
u
e
sin
2
0
g
R


241. A point is moving in a circle. The ratio of its angular velocity about a point on thecircumference of the circle
and about the centre of circle is
a) 1 : 2 b) 2 : 1 c) 1 : t d) t : 1

242. A cylindrical drum is pushed along by a board of length l. The drum rolls forward on the ground a distance of
l/2. There is no slipping at any instant. During the process of pushing the board, the distance moved by the man
on the ground is

a) l/2 b) 3 l/2 c) l d) None of these

243. A string is wrapped around a cylinder of mass M and radius R. The string is pulled vertically upward to
prevent the centre of mass from falling as the cylinder unwinds the string. Find the length of the string
unwound when the cylinder has reached a speed e .
a)
g
R
2 2
e
b)
g
R
2
2 2
e
c)
g
R
4
2 2
e
d)
g
R
8
2 2
e


244. A uniform ball of radius r rolls without slipping down from the top of sphere of radius R. The angular velocity
of the ball at the moment it breaks off the sphere will be (beglect initial velocity of ball)
a)
( )
2
17
10
r
g r R +
b)
( )
2
10
7
r
g r R +
c)
( )
2
5
r
g r R +
d)
( )
2
5
2
r
g r R +

QUESTION BANK
59

245. A smooth rod of length l is kept inside a trolley at an angle u as shown in the figure. What should be the
acceleration a of the trolley so that the rod remains in equilibrium with respect to it?


246. A solid body rotates about a stationary axis according to the law
3
2 6 t t = u . What is the mean value of
angular velocity over the time interval between t = 0 and the time when the body comes to rest?
a) 2 rad/s b) 7 rad/s c) 3 rad/s d) 4 rad/s

247. A solid sphere of radius R is pulled by a force F acting at the top of the sphere as shown in the fig. There is no
slipping anywhere. Work done by force F, when the centre of mass moves a distance s is

a) Fs b) 2 Fs c) Zero d)
2
3
Fs

248. The plank in the figure moves a distance 100mm to the right while the centre of mass of the sphere of radius
150mm moves distance 75mm to the left. The angular displacement of the sphere (in rad) is (assume there is
no slipping)

a)
6
1
b)
6
7
c) 1 d)
2
1


249. A uniform rod of mass m is bent into the from of a semicircle of radius R. The M.I. of the rod about an axis
passing through A and perpendicular the the plane of the paper is
QUESTION BANK
60


250. Let be the M.I. of a uniform squire plate about an axis AB that passes through its centre and is parallel to two
of its edges. CD is a line in the plane of the plate that passes through the centre of plate and makes an angle u
with AB. The M.I. of the plate about the axis CD is then equal to
a) I b) I u
2
sin c) I u
2
cos d) I 2 / cos
2
u

251. From the circular disc of radius R and mass M a concentric circular disc of small radius r is cut and removed,
the mass of which is m. The M.I. of the angular disc remaining about its axis perpendicular to the plane and
passing through the centre of mass will be
a) ( )
2 2
2
1
r R M + b) ( )( )
2 2
2
1
r R m + c) ( )( )
2 2
2
1
r R m d) ( )
2 2
2
1
mr MR

252. A mass M is moving with a constant velocity parallel to the x-axis. Its angular momentum with respect to
origin.
a) is zero b) remains constant c) goes on increasing d) goes on decreasing

253. A rod is hanging from a support. Find out the distance x from the point of suspension where force F (impulsive
force) is applied, so that no reaction force is applied by suppot.

a)
3
L
b)
3
2L
c)
4
L
d)
4
3L


254. A spherical solid body of mass m slips without friction down an inclined plane and reaches its bottom with
velocity v. Had this body been rolling down, then its velocity at the bottom would be
a) v
|
.
|

\
|
5
2
b) 2 v c)
2
1
v d)
7
10
v
QUESTION BANK
61

255. A hoop rolls on a horizontal ground without slipping with speed v. Speed of particle P on the circumference of
the hoop at angle u as shown in figure is

a) 2 / sin 2 u v b) 2 / sinu v c) 2 / cos 2 u v d) 2 / cosu v

A Level (Multiple choice Questions)
256. A particle is moving in x-y plane. At certain instant of time, the components of its velocity and acceleration as
follows:
2 2
/ 1 , / 4 , / 3 s m a s m v s m v
y x
= = = . The note of change of speed of this moment is
a)
2
/ 10 s m b) 4 m/s
2
c)
2
/ 5 s m d) 2 m/s
2


257. Block A and C start form rest and move to the right with acceleration a
A
= 12t m/s
2
and a
C
= 3 m/s
2
.
Here t is in seconds. The time when block B again comes to rest is

a) 2s b) 1s c) s
2
3
d) s
2
1


258. An elastic spring has a length l
1
, when tension in it 4N. Its length is l
2
when tension in it is 5N. What will be its
length when tension in it is 9N?
a) 5l
1
4l
2
b) 5l
2
4l
1
c) 4l
1
+ 5l
2
d) 4l
2
+ 5l
1


259. Acceleration vs time graph of a particle moving in a straight line is a shown. If initially particle was at rest,
then corresponding KE vs time graph will be

QUESTION BANK
62
a) b) c) d)

260. Power supplied to a particle of mass 2kg varies with time as
2
3
2
t
P = watt, here t is in sec. If velocity of
particle at t = 0 is v = 0. The velocity of particle at time t = 2sec will be
a) 1 m/s b) 4 m/s c) 2 m/s d) 2 2 m/s

261. Two billiard balls of the same size and mass are in contact on a billiard table. A third ball of the same size and
mass strikes them symmetrically and remains at rest after the impact. The coefficient of restitution between the
balls is
a) 1/2 b) 3/2 c) 2/3 d) 3/4

262. A constant power is supplied to a rotating disc. Angular velocity ( e ) of disc varies with number of rotations
(n) made by the disc as
a)
3 / 1
n e b)
2 / 3
n e c)
3 / 2
n e d)
2
n e

263. Acceleration displacement graph of a particle executing SHM is as shown in fig. The time period of its
oscillation is (in see)

a) 2 / t b) t 2 c) t d) 4 / t

264. Two sinusoidal waves of freq. f an f + f A traveling in same direction interfere with each other and
produce maxima at x = 0 and t = 0. At what time they again give maxima at x = 0?
a)
f A
1
b)
f A 2
1
c)
f A 3
1
d)
f A
2


QUESTION BANK
63
265. Two particles each of mass m and charge q are attached to the ends of a light rod of length 2R. The rod is
rotated at constant angular speed about a perpendicular axis passing through its centre. The ratio of the
magnitudes of the magnetic of the system and its angular momentum about the centre of rod is
a) m q 2 / b) m q / c) m q / 2 d) m q t /

266. Three gases He, O
2
and CH
4
undergo adiabatic process. The variation of temperature with volume is
a) b) c) d)

MECHANICS QUESTION BANK
1. In each of the following questions, find the unknown velocity / velocities. Assume that the thread is
inextensible and taut in each case.
a) b) c)

d) e)

QUESTION BANK
64
f) g) h)


i) j)


k) l)
QUESTION BANK
65

m) n)

o) p)

2. If the end of the cable at A is pulled down with a speed of 2 m/s, determine the speed at which block B rises.



3. If the end of the cable at A is pulled down with a speed of 2m/s, determine the speed at which block B rises.
QUESTION BANK
66


4. If the hydraulic cylinder at H draws in rod BC at 2 ft/s determine the speed of the slider at A.


5. The hoist is used lift the load at D. If the end A of the chain is traveling downward at v
A
= 5ft/s and the end B
is traveling upward at v
B
= 2ft/s, determine the velocity of the load at D.


6. If the end A of cable is moving upwards at v
A
= 14m/s, determine the speed of block B.
QUESTION BANK
67


7. Determine the velocity of block B at the instant when the velocity of block A is 16 in/s, directed upwards.


8. Block C is moving up at the constant speed of 6 in/s. Given that the elevations of blocks A and B are always
equal, determine the velocity of B.


9. The pulley arrangement shown is designed for hosing materials. If BC remains fixed while the plunger P is
pushed downward with a speed of 1 m/s, determine the speed of the load at A.
QUESTION BANK
68


10. In each of the following cases, some blocks are attached to some motor/s through ideal threads and pullies. The
motors are giving out/taking in the threads at the rates shown in the figure. Find the unknown velocity in each
case. The blocks are moving such that threads are always taut.
a) b)

c) d) f)

g)

11. Determine the time needed for the load at B to attain a speed of 8 m/s, starting from rest, if the cable is drawn
into the motor with an acceleration of 0.2 m/s
2
.
QUESTION BANK
69


12. Determine the constant speed at which the cable at A must be drawn in by the motor in order to raise the load
6m in 1.5s.


13. Staring form rest, the cable can be wound onto the drum of the motor at a rate of v
A
= (3t
2
) m/s, where t is in
seconds. Determine the time needed to lift the load 7m. [use of the figure of previous problem]


14. The cylinder C is being lifted using the cable and pully system as shown. If point A on the cable is being
drawn toward the drum with a speed of 2m/s, determine the speed of the cylinder.


15. The cylinder C can be lifted with a maximum acceleration of a
c
=3 m/s
2
without causing the cables to fail.
Determine the speed at which point A is moving toward the drum when S = 4m if the cylinder is lifted from
rest. In the shortest time possible. [use of the figure of previous problem]
QUESTION BANK
70

16. The motor at C pulls in the cable with an acceleration a
C
= (3t
2
) m/s
2
, where t is in second. The motor at D
drawn in its cable at a
D
= 5m/s
2
. If both motors start at the same instant form rest when d = 3m, determine (a)
the time needed for d = 0 and (b) the relative velocity of block A with respect to block B when this occur.


17. The motor draws in the cable at C with a constant velocity of v
C
= 4m/s. The motor draws in the cable at D
with a constant acceleration of a
D
= 8m/s
2
. If v
D
= 0 when t = 0, determine (a) the time needed for block A to
rise 3m, and (b) the relative velocity of block A with respect to block B when this occurs


18. If motors at A and B draw in their attached cables with an acceleration of a = (0.2t) m/s
2
, where i is in seconds,
determine the speed of the block when it reaches a height of h = 4m, starting from rest. Also, how much time
does it take to reach this height?

QUESTION BANK
71

19. The mine car C is being pulled up the incline using the motor M and the rope-and-pulley arrangement shown.
Determine the speed v
p
at which a point P on the cable must be traveling toward the motor to move the car up
the plane with a constant speed of v = 2m/s.


20. Each of the following problems shows some blocks attached to each other through ideal strings. Find the
unknown velocity at the instant shown in the figure.
a) b) c)

d) e) f)

QUESTION BANK
72
g) h) i)
j)

21. A girl flies a kite at a height of 300ft, the wind carrying the kite horizontally away from her at a rate of 25
ft/sec. How fast must she let out the string when the kite is 500 ft away form her?

22. In each of the following questions, find the value of unknown velocity V at the instant shown in each figure.
Assume that all bodies are perfectly rigid and that they are moving without any rotation.
a) b) c)

d) e) f)

23. Two wedges A and B are being moved with constant velocities V
1
and V
2
as shown in the figure. Find the x
and y components of the block C. Assume that C neither rotates nor loses contact with A and B.
QUESTION BANK
73

24. Block A is being moved with a constant speed V
0
towards left. Find the x y components of velocity of sphere
at the instant x = R. Assume that the sphere moves without rotating.


25. Find the speed of sphere C at instant R x 3 2 = . Where R is the radius of each sphere. Assume that all the
spheres are always in contact and that they move without rotating.


26. Each of the following problems shows an arrangement of identical frictionless spheres placed on a horizontal
table. Each sphere is moving such that it is always in contact with neighboring spheres. Find the unknown
velocities at the instant shown in each figure.
a) b) c)

QUESTION BANK
74
d)
27. In each of the arrangements shown on next page, some particles are connected to each other via ideal threads
and pullies. Some of the pullies are step pullies. Find the magnitude and direction of unknown velocity in each
case.
a) b) c)

d) e)
f) g)

QUESTION BANK
75
h) i) j)

k)

28. In the given arrangement find x and y components of velocity of rod B at the instant rod A makes an angle u
with vertical. Rod A is being rotated at constant angular speed e and length of each rod is l.




29. In the given arrangement, find the speed of block C at the instant shown in the figure.
QUESTION BANK
76


30. In the given arrangement, block A is being moved downward with a constant speed V
A
. At the instant shown in
figure, find i) speed of B ii) Angular speed of rod AB.


31. Point B of bar BD is being pushed to the right with constant velocity V
A
. At the instant shown in the figure.
find
i) angular speed of rod BD ii) velocity of point B iii) speed of point D


32. The wheel is rolling without slipping. Its centre has a constant velocity of 0.6m/s to the left. Compute the
angular velocity of bar BD and the velocity of end D when u = 0.

33. A block A is attached to the rim of a circular disk as shown in the figure. The wheel is rotating with a constant
angular speed e . Find the velocity of block A at the instant shown in the figure.


34. In the given arrangement, find the speed of point C at the instant shown in figure.
QUESTION BANK
77









KEY
1) a) -1m/s, b) 14m/s, c) 2m/s, d) 10m/s, e) 8m/s, f) 5m/s, g) 14m/s, h) 4/3 m/s, i) 7/4 m/s, j) 12m/s
k) 16m/s, l) 4m/s, m) 12m/s, n) 10m/s, o) 10m/s, p) 2m/s
2)
0.5m/s |
3)
0.5m/s |
4)
4 ft/s +
5) 6) 2
m/
s
|

7)
32 in/s +
8)
4 in/s +
9)
10) a) 2m/s, b) 4m/s, c) 6m/s, d) 1m/s, e) 6 m/s, f) 8m/s, g) 6m/s
11) 160s 12)
32 m/s +
13) 3.83s 14) 0.667
m/s |
15)
16) 1.07s, 5.93 m/s 17) a) 1.22s,
b) 2.90m/s |
18) 19) 6 m/s


20)
a) s m/
3
20
, b) s m/ 2 5 , c) s m/ sec 10 u , d) s m/
cos
cos 10
2
1
u
u
, e) s m/ cot 10 u , f)
( )
s m/
cos
cos 10
2 1
2
u u
u

,
g) ( ) s m ec / cos 1 4 u , h)
2 1
tan 10 u = V , s m V /
tan
tan 10
1
2
1
u
u
= , i) s m V s m V / cos 10 , / 10
2 1
u = = ,
j)
( )
s m/
cos 1
10
u +


QUESTION BANK
78
21) 20 ft /sec 22)
a) s m/
3
10
, b) s m/ 3 10 , c) 5m/s, d) s m/
3
10
, e) s m/ 3 10 f) 5 m/s
23)
( )
( )
u
u
sin
2
, sin
2
2 1
1 2
V V
V
V V
V
y
x
+
=

=

24)
,
tan 2
,
2
0 0
u
V
V
V
V
y x
= =

25)
V V
V
V
y x
,
2
3 3
,
4
3
= =


26)
a) V = 10/s, b) s m/ 3 5 , c) s m V V
C B
/ 3 5 = = , d) s m V s m V
B A
/ 5 . 7 , / 3 5 = =
27. a) 5m/s, b) 2.5m/s, c) 40/9 m/s, d) 15m/s, e) 20/9 m/s, f) 65/6 m/s, g) 16 m/s, h) Thread cannot remain taut. Hence, B will fall down with
acceleration g.
28)
( ), cos rightward r V
x
u e =

( ), cos downward r V
y
u e =

29)
righwards l u ecos 2

30) i)
rightwards t V V
A B
u e =

ii)
u
e
sin i
V
A
=
iii)
u
3
2
sin L
V
a
A
B
=
rightwards

33)
3
3 2 +
= e r V
(rightwards)

















QUESTION BANK
79








Multiple choice questions (only one is correct)
1. A body of specific gravity 6, weighs 0.9kg when placed in one pan (say pan A) and 1.6kg when placed on the
other pan (pan B) of a false balance. The beam is horizontal when both the pans are empty. Now if the body is
suspended form pan A and fully immersed in water, it will weigh
a) 0.5kg b) 0.6kg c) 0.75kg d) 0.8kg

2. The pulley has mass M >m. String is massless. The above system is released from rest from the position
shown. Then

a) Body (1) will slowly come down, till equilibrium is attained at level AA shown.
b) The system will perform oscillations with equilibrium position at level AA and amplitude
0.5m
c) The response of the system depends on whether pulley-string interface has friction or not.
d) The system will continue to be in the same initial position.

3. In the position shown above Let T
1
be the tension in the left part of the string, T
2
= Tension in the right part, N
= Normal reaction on block (2) by the resting surface. Then
QUESTION BANK
80


a) T
1
= T
2
= 0 and N = mg b) T
1
=mg, T
2
= 0, N =mg
c) T
1
= T
2
= mg and N = 0 d) T
1
= T
2
=
2
mg
and N=
2
mg


4. A body of mass m was slowly hauled up the hill and down the hill onto the other side by a force

F which at
each point was directed along a tangent to the trajectory. The work performed by this force, if the coefficient of
friction is
1
uphill and
2
dowbhill, is

a) mg ( )
2 2 1 1
+ b) mg ( )
2 2 1 1
2 + + h
c) mg ( )
2 2 1 1
+ + h d) mg ( )
2 2 1 1


5. The bob of a pendulum is taken to position A and given an initial velocity u in the direction shown, in the
following cases.

If the minimum value of u so that the pendulum reaches position OB in case (i) and position OC in case (ii) are
u
1
in case (i) & u
2
in case (ii), then ( )
2
1
2
10

= ms g is
u
u

QUESTION BANK
81
a) ( ) 2 3 + b) ( ) 3 2+ c) ( ) 3 2+ d) ( ) 2 2 3

6. A conveyor belt carrying powdery material is at an angle 37
0
to the horizontal and moves at a constant speed of
1ms
-1
as shown. Through a small hole in the belt, the powdery material drops down at a constant rate of 1kg
per second. What is the force to be applied on the belt along the direction of its motion so as to maintain its
constant speed of 1ms
-1
?

a) -1 N b) Zero c) +1 N d) None of the above


7. Small body A on a hemispherical body B which is on a wedge C which is on a smooth horizontal surface.
System is released from rest from the position shown when cm cm
B
5 , 10 , 37
0
= = = o .
When
C
, 53
0
= o is 4.5cm
B
at that instant is (neglect friction)

a) 9 cm b) 10 cm c) 10.5 cm d) 11 cm

8. A pendulum consists of a mass m attached to the end of a light string 0.5m long. It can oscillate in the vertical
plane. If it is let go in the horizontal position and has an inelastic collision with the floor, e = 0.5, the rebound
velocity is (ms
-1
) (the angle turned is 6 / t ) (g = 10ms
-2
)

QUESTION BANK
82
a)
2
5
b)
4
5
c)
4
35
d)
4
45


9. The track is in the vertical plane. The track is rough with friction coefficient . A particle at A is allowed to
slide down and goes upto B and returns. Heights of A and B are 0.2m and 0.1m respectively, as shown. The
maximum possible value of is

a)
4
1
b)
7
1
c)
6
1
d)
5
1


10. A particle is projected form a point on a horizontal floor. After it has three collisions with the floor, it is found
that the ratio of maximum height to minimum peaks reached by it is
|
|
.
|

\
|
36
12
2
10
. The coefficient of restitution is
a) .5 b) 0.64 c) 0.8 d) 0.9


11. The potential energy function along the positive x axis is given by ( )
x
b
ax x U + = , a, b are constants. If it is
known that the system has only one stable equilibrium configuration, the possible values of a and b are
a) a = 1, b = 2 b) a = 1, b = -2 c) a = -1, b = 2 d) a = -1, b = -2

12. The acceleration of the 1kg block immediately after the string is cut is (g = 10ms
-2
).

QUESTION BANK
83
a) 4ms
-2
b) 4.1ms
-2
c) 16ms
-2
d) 40ms
-2


13. A circular pan with its side wall inclined inward at 53
0
as shown is rotating about its central vertical axis with a
constant angular velocity. A ball placed at the edge rotates along with it as shown. If the ball exerts a force of
22.5 N on the side wall and 23.5N on the bottom surface, the mass of the ball is

a) 1kg b) 2kg c) 3kg 4kg

14. Acceleration of 10kg block, when system released from rest, is

a) 0.5ms
-2
b) 1 ms
-2
c) 1.25 ms
-2
d) 1.66ms
-2


15. Case(A)

Case (B)

The reaction force between the 5kg and 6kg block in case A and case B will be
a) equal and non zero b) unequal with case A being more
c) unequal with case being more d) equal and zero

For Question No. 16 to 20:
Each question consider of two statements: one is Assertion (A) and the other is Reason(R). You are to examine
these two statements and select the answer using the code given below.
a) Both A and R individually correct, but R is the correct explanation of A
b) Both A and R individually correct, but R is the not correct explanation of A
c) A is true but R is false d) A is false but R is true
QUESTION BANK
84
16. Assertion (A): If a block is released form rest, when reaches the bottom point of the wedge, its
speed is same irrespective whether the wedge is fixed or the wedge is fee to move.
Reason(R): Mechanical energy is conserved in both cases.

17. Assertion (A): A body is at rest on floor. You lift it vertically up and bring it to rest at a point h
above the ground. The work done by you is zero.
Reason(R): Any non-zero work done by a force on a body results in change in kinetic energy of
the body.

18. Assertion (A): The negative of the work done by the conservative internal forces on s system
equals to change in its potential energy.
Reason(R): Work energy theorem.

19. Assertion (A): In a tug of war that team wins which applies more tension force on string then
their opponents.
Reason(R): The winning team must be having stronger players.

20. Assertion (A): If a block starts moving at t = 0 with an acceleration a then its kinetic energy at
time t with respect to two non-inertial frames which have same acceleration in a
direction is not always same.
Reason(R): The work energy theorem W K A A is not for non-inertial frame.

Match the following Questions 21 to 25
21. Two columns are given in each question. Match the elements of Column-I with Column-II.
Column-I Column-II
i) Friction force p) Contact force
ii) Normal reaction q) Electromagnetic force
iii) Tension in a string r) Gravitational force
iv) Force between two charges of mass m s) Nuclear force
a) (i-p,q), (ii-p,q), (iii-q,s), (iv-q,r) b) (i-p,q), (ii-p,q), (iii-q), (iv-q,r)
c) (i-p,q), (ii-p), (iii-q,s), (iv-q) d) (i-q), (ii-p), (iii-q,s), (iv-p,q,r)

22. In Column-I there are some motions of a body and Column-II contains the list of concepts that can be used for
the analysis of these motions
Column-I Column-II
QUESTION BANK
85
i) A body moving in a vertical circle p) Conservation of energy
ii) A body moving in a horizontal circle q) Conservation of momentum
iii) A body dropped form a height on a block attached
to the top of a vertical spring r) Centripetal force
iv) Rocket propulsion s) Centrifugal force
a) (i-p,q,r), (ii-q,r,s), (iii-p,q,s), (iv-p) b) (i-p,s), (ii-p,r), (iii-q), (iv-p,q,r)
c) (i-p,r), (ii-q,s), (iii-p), (iv-p,q) d) (i-p,r,s), (ii-r,s), (iii-p,q), (iv-q)

23. When two bodies collide they come in contact at t = t
1
and loses contact at t = t
2
. This (t
2
-t
1
) is a very small
time interval. Consider this small time interval and match the following.
i) Elastic collision p) Kinetic energy decreases and potential
energy increases and then potential
energy decreases and kinetic energy increases
ii) Inelastic collision q) Kinetic energy + potential energy is conserved
iii) Perfectly inelastic collision r) Momentum is conserved
iv)Oblique elastic collision s) Kinetic energy decreases and potential energy
increases and then situation remains same
a) (i-p,q,r), (ii-q,r,s), (iii-p,q,s), (iv-p) b) (i-p,s), (ii-p,r), (iii-q), (iv-p,q,r)
c) (i-p,r), (ii-q,s), (iii-p), (iv-p,q) d) (i-p,r,s), (ii-r,s), (iii-p,q), (iv-q)

24. For each of the following four cases of Column-I match the range of force F in column B so that the block m is
not slipping on the surface with which it has contact.
For all cases m = 2kg; 2 . 0 = Take g = 10m/s
2

Column-I Column-II
p) (48N, 80N)
q)
|
.
|

\
|
N N
11
500
,
19
500

QUESTION BANK
86

r)
|
.
|

\
|
N N
11
380
,
19
220

s)
|
.
|

\
|
N N
17
380
,
23
220

a) (i-p), (ii-q), (iii-q,r), (iv-p,q,r,s) b) (i-p), (ii-q), (iii-r,s), (iv-q,r)
c) (i-q), (ii-p,s), (iii-q,s), (iv-r,s) d) (i-q,r), (ii-p,q), (iii-r), (iv-p,q,r)

25. A body is moving in a vertical circle of radius R, considering motion from top point of circle bottom most
point of circle, match the following.
Column-I Column-II
i) Tangential accelertaion p) Always increases
ii) Centripetal acceleration q) Always decreases
iii) Angular velocity r) First increases then decreases
iv) Potential energy s) Variable depending on angle made with
vertical by string
t) Variable independent of the made angle with
vertical by string
a) (i-r,s), (ii-q,r), (iii-p,t), (iv-p,q) b) (i-q,s), (ii-r,s), (iii-p,s), (iv-q,r)
c) (i-p,s), (ii-q,r), (iii-q,s), (iv-p,r) d) (i-r,s), (ii-q,s), (iii-p,s), (iv-p,s)

Write the final answer to each question in this section in the column provided.
26. The block ABCDE of mass 5m has BC part spherical, of radius 1m, is on frictionless horizontal surface. BC is
quarter circle. A small mass m is released at B and slides down. How far away form D does it hit the floor? (g
= 10ms
-2
)
QUESTION BANK
87


27. The kinetic energy of a particle of mass 1kg moving along a circle of radius 1m depends on time t as K = t
4
.
Fid the force acting on the particle as function of t.


28. Spring is already in a compressed potion with initial compression = 5cm. The 10kg block is allowed to fall.
Determine the maximum compression of the spring before the block rebounds. (g = 10ms
-2
).


29. A long plank of mass M = 8kg, length m 1 = rests on a horizontal surface. Coefficient of friction 2 . 0
1
= .
A small block, mass m = 2kg rests on the right extreme and of M, on the rough top surface. At t = 0, a force F=
25.5N is applied on M towards the right. (see figure). The block m does not slide on M. At t = 3s, force F is
increased to 31N. The block m falls off Ms surface at t =7s. Determine the coefficient of friction between m
and M.


30. A particle is released on the smooth inside wall of a cylindrical tank at A with a velocity u which makes an
angle o with the horizontal tangent. When the particle reaches a point B, a distance h below A, determine the
angle | (as an inverse function of cosine) made by its velocity with the horizontal tangent at B.
QUESTION BANK
88

Passage-I (31 to 33):
Two blocks of masses 10kg and 5kg are placed on a rough horizontal floor as shoen in figure. The strings and
pulley are light and pulley is frectionless. The coefficient of friction between 10kg block and surface is 0.3
while that between 5kg block and surface is 0.2. A time varying horizontal force. P=5t Newton (t is in sec) in
applied on 5kg block as known. [Take g=10ms
-2
]



31. The motion of block starts at t = t
0
, then t
0
is
a) 14s b) 8s c) 9s d) 12s

32. The friction force between 10kg block and surface at
2
0
t
t = is in between
a) zero and 10N b) 10N and 35N c) 12.5N and 17.5N d) 12.5N certain value

33. The acceleration of 5kg block at t = 2t
0
is
a) 12 ms
-2
b)
2
5
14

ms c)
2
9
14

ms d) 2 ms
-2


Passage-II (34 to 36):
A block of mass 4kg is pressed against a rough wall by two perpendicular horizontal force F
1
and F
2
as
coefficient of static friction between the block and floor is 0.6 and that of kinetic friction is 0.5. [Take g=10ms
-
2
]

QUESTION BANK
89

34. For F
1
= 300N and F
2
= 100N, find the direction and magnitude of friction force acting on the block.
a) 180N, vertically upwards b) 40N, vertically upwards
c) 107.7N, making an angle of
|
.
|

\
|

5
2
tan
1
with the horizontal in upward direction.
d) 91.6N, making an angle of
|
.
|

\
|

5
2
tan
1
with the horizontal in upward direction.

35. For F
1
=150N and F
2
=100N, find the direction and magnitude of friction force action on block.
a) 90N, making an angle of
|
.
|

\
|

5
2
tan
1
with the horizontal in upwards direction
b) 75N, making an angle of
|
.
|

\
|

5
2
tan
1
with the horizontal in upwards direction
c) 170.7N, making an angle of
|
.
|

\
|

5
2
tan
1
with the horizontal in upwards direction
d) Zero

36. For data of Question No.35, find the magnitude of acceleration of block.
a) Zero b) 22.5 ms
-2
c) 26.925 ms
-2
d) 8.175 ms
-2


37. System shown in figure is in equilibrium and at rest. The spring and string are massless, now the string is cut.
The acceleration of mass 2m and m just after the string is cut will be

QUESTION BANK
90
a)
2
g
upwards, g downwards b) g upwards,
2
g
downwards
c) g upwards, 2g downwards c) 2g upwards, g downwards










KEY
1) c 2) d 3) c 4) a 5) d
6) b 7) c 8) c 9) b 10) b
11) c 12) d 13) a 14) d 15) c
16) d 17) d 18) c 19) d 20) c
21) b 22) d 23) c 24) a 25) d
26)
m
5
24

27)
N t
6
2+ |
28)
( )cm 1 17
2
5

29)
1 . 0
2
=
30)
|
|
|
|
.
|

\
|
+

2
1
2
1
cos
cos
u
gh
u o

31) a 32) c 33) c 34) c 35) b
36) d 37) a

Solutions
1. i)


2 2 1 1
m m = .(1)

QUESTION BANK
91
ii)


2 1 2 2 2 2 1 1
9 . 0 9 . 0 = + = + M m M m (2)

iii)


2 2 2 1 1 1
6 . 1 M m m + = +

2 1
6 . 1 M = .(3




iv)


2
1
2 1 2 2 2 1 1 1

= = + = + x x x m m
From (2)
2 1
9 . 0 = M
75 . 0
4
3
2 . 1
9 . 0
9 . 0 2 . 1
2
1
2 1
= = = =


kg x 75 . 0 =
QUESTION BANK
92

2. (d) Self explanatory [acc = 0, u = 0 at rest]

3. (c) Self explanatory [acc = 0, u = 0 at rest]

4. Uphill: F = u u cos sin
1
mg mg +
F.ds = u u cos sin
1
ds mg ds mg +

} } }
+ = dx mg dy mg ds F
1
.

1 1
. mg h mg + = (1)
Downhill:
u u cos sin
2
mg mg F =

}
+ = =
2 2
. . mg mgh ds F Fds ds F .(2)
( ) ( ) ( )
2 2 1 1
2 1 + = + = mg W

5. Case (i)

gR u mg
R
mu
T
C
A
> = >
2
1
2
45 cos 0 ..(1)
Case (ii)

gR v mg
R
mv
T
C
C
C
> > >
2
2
0
|
.
|

\
|
+ =
2
2
1
1
2
1
2
1
2
mgR mv mu
C

gR gR gR u gR gR u v
C
> + + = 2 2 2 2
2 2 2

QUESTION BANK
93
( )gR u 2 3
2
2
= .(2)

( )
( )
( ) 2 2 3 2 2 3
2 / 1
2 3
1
2
1
2
2
1
2
2
= =

= =
u
u
u
u


6. 0 . . = =
rel rel reaction
v But v
dt
dm
F .

7. No external force horizontally,

CM
x does not shift
Taking
( ) m x
B A
06 . 0 37 sin 53 sin 3 . 0
/
= =
=
C B
x
/
unknown
m given x
C
005 . 0 = =
( ) ( ) ( ) 0
/ / /
= + + + + +
C C C C B B C C B B A A
x m x x m x x x m
( ) ( ) ( ) 0 005 . 0 3 005 . 0 2 0005 06 . 0 1
/ /
= + + +
C B C B
x x

C C B B C B
x x x x + = =
/ /
01 . 0
. 005 . 0 005 . 0 01 . 0 m = =
cm m
B
5 . 10 105 . 0 05 . 0 10 . 0 = = + =

8. Before collision:
Using loss of P.E = Gain of KE

Components
2
3
. 5 =
y
u

2
5
=
x
u
After collision
QUESTION BANK
94

1
4
15
.

+ = = ms u e v
y y


1 2 2
4
35
2
5

= + = = = ms v v v u v
x y x x


9. Work done by friction, w
f
.
We know w
f
has to be minimum

( ) mg mg , 7 . 0 3 . 0 4 . 0 = + = .
But w
f
= loss of P.E = 0.1mg

7
1
7 . 0 1 < > mg mg



10.


^ ^
cos sin i u j u u u u + =

^ ^
1
cos sin i u j eu u u u + =

^ ^
2
2
cos sin i u j u e u u u + =

^ ^
3
3
cos sin i u j u e u u u + =
H e
g
u e
H
g
e
H
6
2 2 6
3
2 2
2
sin
2
sin
= = =
u u


6
2
6
12
36
6
36
12
6
3
10
2
10
2
2
10 1
|
|
.
|

\
|
= = = = e
e H
H

QUESTION BANK
95
64 . 0
100
64
10
2
2
6
= = = e

11.
( )
( )
2
x
b
a
dx
x dU
+ =
0 0
2
2
=

=
x
b ax
um equillibri
dx
dU


a
b
x and x = =
2
0
b and a are of opposite signs.
options (b) or (c) possible
For stable equation.

2
2
dx
U d
should be +ve
( )( ) ( ) 0 0 0 . 2
3 3
> > > = b x for
x
b
x
b


12. T =70N, T = T -30N=40N
On cutting thread,

1
1
40
1
40

= = ms A
kg

All others acc = 0


13. mg N N + = 37 sin
2 1
..(1)
R m N
2
2
37 cos e = ...(2)
QUESTION BANK
96

5 . 23
1
= N
5 . 22
2
= N
( ) mg 6 . 0 5 . 22 5 . 23 1 =
kg m N mg 1 10 5 . 13 5 . 23 = = =

14.

N f
static
20 =
2
10
20
= = A

5
20 25
5
25
=

=
f
a
< A a not possible
Hence no relative motion

2
66 . 1
5 10
25

=
+
= = ms a A




15. Case(A)

QUESTION BANK
97
a R a R 40 1 40 1
1 1
= =
Case (B)

a R a R 15 1 15 1
2 2
= =

1 2
R R >

16. R is clearly true, but A is false. Initial energy is P.E. If wedge is free to move, it will have KEKE of block
will be less than that if wedgeis fixed.

17. You did positive work and gave positive energy. Gravity did equal negative work and made net KE A zero.

18. Self explanatory.

19. Tension in rope remains same through out but it is the frictional force provided by the ground that helps a team
to win.

20. Assertion is right because only the charge in kinetic energy in both the frames are same (including work of
pseudo force) but k
f
is dependent on the k
i
as
k
f
k
i
= k A
and k
i
depends on the velocities of these reference frames at time t =0 reason is wrong
so assertion is right and reason is wrong

21. (b) self explanatory

22. (d) self explanatory

23. (c) self explanatory

24. (a) 1
st
case
QUESTION BANK
98

uN f s = 37 cos 20 (for no motion)
( ) 37 sin 20 20 2 . 0 16 + + s F
N F 48 >
( ) e , 48 F .(1)
II
nd
case

N f F mg s = 37 sin (to prevent downward slipping)
( ) 37 cos 2 . 0
5
3
20 F
F
s
20
5
3
25
4
>
|
.
|

\
|
+ F
N F
19
500
>
N f mg F s = 37 sin (to prevent upward slipping)

|
.
|

\
|
s
5
4
2 . 0 20
5
3
F
F

20
25
4
5
3
s
|
.
|

\
|
F

|
.
|

\
|
e s N N F so N F
11
500
,
19
500
11
500
..(2)
III
rd
case

QUESTION BANK
99
N f F mg s = 37 sin 37 sin (for no downward slipping)

|
|
.
|

\
|
+
|
.
|

\
|
s
|
.
|

\
|
16
5
4
2 . 0
5
3
12 F F

5
16
12
5
3
25
4
>
|
.
|

\
|
+ F
N F
19
220
19
25
5
44
= >
N f F mg s = + 37 sin 37 sin (for no upward slipping)

|
|
.
|

\
|
+
|
.
|

\
|
s 16
5
4
2 . 0 12
5
3
F
F

N F F
11
380
11
25
5
76
5
76
25
4
5
3
= s s
|
.
|

\
|

So,
|
.
|

\
|
e
11
380
,
19
220
N F (3)
IV
th
Case

N F mg f s = 37 cos 37 sin (for no downward sliding with respect to wedge)

|
.
|

\
|
+ s
5
3
16 2 . 0
5
4
12 F
F

N F F
23
220
5
16
12
25
3
5
4
> >
|
.
|

\
|
+
N f mg F s = 37 sin 37 cos (for no upward sliding with respect to wedge)

|
.
|

\
|
+ s
5
3
16 2 . 0 12
5
4 F
F

17
380
s F

|
.
|

\
|
e N N F
17
380
,
19
220
.(4)

QUESTION BANK
100
25. (d) Self explanatory

26. When m leaves C, let its velocity (horizontal) be u.
Then block 5m will have velocity
5
u
to left.
Energy equation:

1
3
50
3
5

= = ms
gR
u
t = time to reach ground s
g
h
5
1
10
1 2 2
=

= =
Distance of hitting point from D =
m t
u
t u t
u
5
24
5
1
.
3
50
5
6
5
6
5
= = = + =

27.
4 2 2
2
1
2
1
t v mv K = = = ..(1)

4 2
2 2
2
1
. 1
t v
v
R
mv
F
n
= = = =

4 2
2t v =
2
2 t v = t
dt
dv
2 2 =
t
dt
dv
dt
dv
m F
t
2 2 . 1 = = =
Resultant
6 2 2
2 2 t t F F F
n t
+ = + =

28. Initial energy:
( ) J kx E P
spring
5 05 . 0 4000
2
1
2
1
.
2 2
= = =
J mgh E P
mass
20 2 . 0 100 . = = =
Total J E
initial
25 =
Final energy
( ) ( )
2 2
' 05 . 0 4000
2
1
2
1
. x x x k E P
spring
+ = + =
QUESTION BANK
101
( ) ' 200 ' 2000 5 ' 1 . 0 ' 0025 . 0 4000
2
1
2 2
x x x x + + + +
' 100 ' . x mgx E P
mass
= =
0 20 100 ' 2000
2
= + x x
( ) ( )cm m x 1 17
2
5
1 17
200
5
' = =

29. From t = 0 to t = 3sec

f
2
< f
2

max

so both blocks are moving together and then
F f
1 max
= (m + M) a (1)
f
1 max
=0.2 x 10 x 10 = 20N
So 25.5 20 = 10a

1
/ 55 . 0

= s m a .(2)
Displacement
2
1
3 55 . 0
2
1
= S
=2.475m .(3)
Velocity = 0.55 x 3
= 1.65m/sec .(4)
From t = 3 to t = 7 sec
f
2
=f
2 max
= 20
2


m
a 8 20 20 31
2
=

m
a 2 20
2
=
( )
2
4
2
1
1
m M
a a =

8
1
=
m M
a a
After solving these equation.
1 . 0 , 1 ,
8
9
2
= = =
m M
a a
QUESTION BANK
102
30. Initial velocity u can be split into tangential component
and u o cos vertical component u sino
v
z
at B can be attained by,
gh u v
z
2 sin
2 2 2
+ = o
Normal reaction
r
mu
N
o
2 2
cos
=
tangential component at o cos u B =
velocity at gh u u v B
z
2 cos
2 2 2 2
+ = + = o
| Z is given by
2
1
2
1
cos
cos
u
gh
u
+

o


31 to 33.
From constraint theory we can relate the acceleration 10kg and 5kg blocks.

Limitng friction force between 5kg blocks and surface is, N f
L
10 10 5 2 . 0
1
= =
Limiting friction force between 10kg block and surface is, N f
L
30 10 10 3 . 0
2
= =
For 5kg block, P-2T - 0
1
= f [in equilibrium, i.e., when blocks are not moving]
For no motion of blocks,
2 1
2 1 L L
f f and f f s >
So,
2 1
2
L L
f T and f T P s s

2 1
2
L L
f f P + s
So, for motion to take place,
2 1
2
L L
f f P + >
30 2 10 2 5
2 1
0
+ = + =
L L
f f t
s t 14
0
=
At s
t
t 7
2
0
= = , the equation for 5kg and 10kg blocks are
0 0 2 35
2 1
= = f T and f T
QUESTION BANK
103

2 1
2 35 f f + =
And we know at t = 7s both the blocks are at rest so

2 1
2 1 L L
f f and f f s s
Solving above equation we get, ; 10 0
1
N f s s
N f N 5 . 17 5 . 12
2
s s
And N T 5 . 17 5 . 12 s s
At t = 2t
0
= 28s, equation for blocks are
140 2T 10 = 5a and T 30 = 10 x 2a

2
9
14

= ms a

34. The forces acting on the block are F
1
, F
2
, mg, normal contact force and frictional force. Here fractional force
wont act along vertical direction as the component of resultant force along the surface acting on body is not
along vertical direction and direction of the friction force is either opposite to the motion of block (direction of
acceleration of block) if it is moving or not moving.

So, N N f
L
180 300 6 . 0
1
= = =
Resultant of 4g and F
2
is 107.7N making an angle of
|
.
|

\
|

5
2
tan
1
with the horizontal
As force applied along the surface is
L
f > , so the block doesnt move and friction is static in nature.
N f 7 . 107 = making an angle of
|
.
|

\
|

5
2
tan
1
with the horizontal in upward direction.

35. For N f N F
L
90 150 6 . 0 , 150
1
= = =
As component of resultant force along the surface is 107.7N and is greater than
L
f , so kinetic friction comes
into existence, i.e., friction force acquires the value . 75 150 5 . 0
1
N N f
k
= = = Its direction is opposite to
component of resultant force along the surface.

QUESTION BANK
104
36. Acceleration of block
2
175 . 8
4
75 7 . 107

=

= ms

37. Initially under equilibrium of mass m: T = mg
Now, the string is cut. Therefore, T = mg force is decreased on mass m upwards and downwards on mass 2m.
g
m
mg
a
m
= = (downwards)
And
2 2
2
g
m
mg
a
m
= = (upwards)


























QUESTION BANK
105













TOPIC: - CONSERVATION OF MOMENTUM AND COLLISIONS

1) A small particle is attached to one of the ends of a light inextensible
string of length 2 m, and placed on a smooth horizontal surface. The
other end of the string is fixed to a wall. If a velocity v = 10 c m/s be
given to the particle, find the final velocity with which the particle
moves, after the string becomes taut.
l = 1m
v = 10cm/s


2) Two particles A and B of mass 100 g and 200 g are attached to the two ends
of a string of length 5 m, resting on a smooth floor. They are separated by a
distance of 3 m. A is given a velocity of v = 20 cm/s, along a direction
normal to AB. Determine the velocities of A and B just after the string
becomes taut?

A
B
V
3 m


3) A particle (a mud pallet, say) of mass M strikes a smooth stationary
wedge of mass M with a velocity V
0
, at an angle u with horizontal. If
collision is perfectly inelastic, find the
a) velocity of the wedge just after the collision
b) Change in K.E of the system (M+m) in collision

m
M
u
0
V


QUESTION BANK
106
4) Two particles of masses
1 2
m and m are connected by a light and
inextensible string which passes over a fixed pulley. Initially, the
particle
1
m moves with a velocity
0
v when the string is not taut.
Neglecting friction in all contacting surfaces, find the velocities of the
particles
1 2
m and m just after the string is taut.
0
V
1
m
2
m
x


5) Three identical particles A, B and C lie on a smooth horizontal table.
Light in extensible strings which are just taut connect AB and BC and
0
ABC is 135 Z . An impulse J is applied to the particle C in the
direction BC. Find the initial speed of each
particle. The mass of each is m.
A B
C
J
1
3
5
0

6) In the movie Aajab Prem ki Ghazab kahani, Say Ranbir kapoor and Katrina kaif each weighing 40 kg are
sitting on a friction less platform some distance d apart. Ranbir rolls a ball of mass 4 kg on the platform
towards Katrina kaif of course she catches it. Then Katrina rolls the ball towards ranbir and he catches it . The
ball keeps on moving back and forth between ranbir and Katrina kaif. The ball has fixed speed of 5 m/s on the
platform.

a) Find the speed of Ranbir kapoor after he rolls the ball for the first time.
b) Find the speed of Ranbir kapoor after he catches the ball for the first time
c) Find the speed of Ranbir kapoor and Katrina kaif after the ball has made 5 round trips
and is held Ranbir kapoor.
d) How many times can Ranbir roll the ball
e) What is the centre of mass of the system (Ranbir + Katrina + ball) at the end of the n
th

trip.










QUESTION BANK
107
















PAPER I
Topic: Projectile Motion

SECITON I (Single Correct Answer)
1. The equation of projectile is
2
qx px y = . Its horizontal ranges is
a) pq b) p/q c) p d) q

2. A large number of bullets are fired in all directions with the same speed V. What is the maximum area on the
ground on which these bullets will spread?
a)
g
V
2
t b)
2
2
g
V
t c)
g
V
2
2
t d)
g
V
2
2
t
3. A particle has an initial velocity of
^ ^
4 3 j i + and an acceleration of
^ ^
4 . 0 3 . 0 j i+ . Its speed after 10s is
a) 10 units b) 8.5 units c) 2 7 units d) 7 units

4. Two paper screens A and B are separated by a distance of 200m. A bullet pierces A and then B. The hole in B
is 40cm below the hole in A. If the bullet is traveling horizontally at the time of hitting A, then the velocity of
the bullet at A is
a) 200 m/s b) 400 m/s c) 600 m/s d) 700 m/s
QUESTION BANK
108

5. A particle starts form the origin of coordinates at time t = 0 and moves in the x-y plane with a constant
acceleration o in the y direction. Its equation of motion is
2
x y | = . Its velocity component in the x-direction
is
a)
|
o
2
b)
|
o
2
c)
|
o 2
d) Variable

6. The taken by the projectile to reach form A to B is t, then the distance AB is equal to

a) t u 2 b) t u 3 c) t u
2
3
d)
3
t u


7. From an inclined plane two particles are projected with same speed at same angle u , one up and other down
the plane as shown in figure. Which of the following statements (s) is /are correct?

a) The time of flight of each particle is the same
b) The particles will collide the plane with same speed
c) Both the particles strike the plane perpendicular
d) The particles will collide in mid air if projected simultaneously and time of flight of each
particle is less than the time of collision

8. A particle moves in the x y plane with velocity 2 2 8 = =
y x
v and t v . If it passes through the point x = 14
at t = 2s, then the equation of the path is
a) 2
2 3
+ = y y x b) 2
2
+ = y y x c) 2 3
2
+ = y y x d) 2 2
2 3
+ = y y x

QUESTION BANK
109
SECITON II (Multiple Correct Choice )
9. A ball is dropped onto a pad at O and rebounds with a velocity u at an angle 60
0
with the horizontal as shown
in Figure. The ball will enter the window W of width 0.5m at a height 1m, if u is

a) 50 m/s b) 10 m/s c) 6 m/s d) 5 m/s

10. The path of a particle moving under the influence of a force fixed in magnitude and direction is
a) Straight line b) Circle c) Parabola d) Ellipse

11. Two projectiles are fired form the same point with the same speed at angle of projection 60
0
and 30
0

respectively. Which one of the following is true?
a) Their maximum height will be the same
b) Their range will be the same
c) Their landing speed will be the same
d) Their time of flight will be the same


12. In case of projectile motion of two projectiles A and B are projected with the same speed at angles 15
0
and 75
0

respectively to the horizontal, then
a) H
A
> H
B
b) H
A
< H
B
c) T
A
> T
B
d) T
A
< T
B


SECITON III (Questions with Comprehension Type)
PASSAG - 13 to 15:
The path of a projectile motion is given by the relation
80
2
x
x y = . Here x and y are in metres and g = 10
m/s
2
. For this projectile answer the following questions:
13. What is the initial velocity of projection of the projectile
a) s m/ 2 10 b) s m/ 20 c) s m/ 2 20 d)
1
/ 2 10

s m

14. The initial angle of projection of the projectile is
QUESTION BANK
110
a) ( ) 1 tan
1
b) ( ) 2 / 1 tan
1
c) ( ) 2 tan
1
d) ( ) 4 tan
1


15. The maximum height of the projectile is
a) 20 m b) 40 m c) 60 m d) 80 m

PASSAG - 16 to 18:
Two second after projection, a projectile is traveling in direction inclined at 30
0
to the horizontal. After 1 more
second, it is traveling horizontally (Use g = 10ms
-2
).
16. The time of flight of projectile is
a) 3 s b) 4 s c) 4.5 s d) 6 s

17. The initial velocity of its projection is
a) 10 m/s b) 3 10 ms c) 20 m/s d) 3 20 ms
-1


18. The angle of projection of the projectile is
a) 30
0
b) 45
0
c) 60
0
d) None of the above

SECITON IV (Match following Type)
19. The equation of motion of a projectile is y = ax bx
2
, where a and b are constant of motion. Match the
quantities of Column-I with the relations of Column-II
Column-I Column-II
A) The initial velocity of projection P) b a/
B)The horizontal rage of projectile Q)
bg
a
2

C) The maximum vertical height attained by projectile R)
b
a
4
2

D) The time of flight of projectile S)
( )
b
a g
2
1
2
+



20. Four balls of same masses are projected with the equal speeds at angles 15
0
, 30
0
, 60
0
and 75
0
with the
horizontal direction from the ground. Match the quantities of Column-I with the angle of projection given in
Column-II
Column-I Column-II
QUESTION BANK
111
A) Horizontal range is maximum for angle P)
0
15
B) The vertical height is maximum for angle Q)
0
30
C) The time of flight is minimum for angle R)
0
60
D) The horizontal range is minimum for angle S)
0
75
KEY

1 b 2 b 3 a 4 d 5 b
6 d 7 a 8 b 9 a,b,c 10 a,c
11 b,c 12 b,d 13 c 14 a 15 a
16 d 17 d 18 c 19
A-p ; B-s,
C-r ; D-q
20
A-q,r ; B-s
C-p ; D-p,s

SOLUTIONS
1. The equation for the path of projectile during oblique projection is

u
u
2 2
2
cos 2
tan
u
x g
x y =

|
|
.
|

\
|
=
u u
u
2 2
cos tan 2
1 tan
u
x g
x

|
|
.
|

\
|
=
u
u
2 sin
1 tan
2
u
x g
x

|
.
|

\
|
=
R
x
x 1 tanu ..(i)
Where R is the horizontal range of projectile. From the given equation of projectile, we have

|
|
.
|

\
|
= x
p
q
x p 1 ..(ii)
Comparing (i) and (ii), we have

q
p
R or
p
q
R
= =
1

2. If R is the max. H.R., then area covered by bullets

2
2
2
|
|
.
|

\
|
= =
g
V
R t t

QUESTION BANK
112
3. 4 3 ; 4 . 0 3 . 0
^ ^
= = + =

y x
u and u so j i u
4 . 0 3 . 0 ; 4 . 0 3 . 0
^ ^
= = + =

y x
a and a so j i a
6 10 3 . 0 3 = + = + = t a u v
x x x

8 10 4 . 0 4 = + = + = t a u v
y y y

So units v v v
y x
10 8 6
2 2 2 2
= + = + =

4. Refer to figure

Or t = 200 / u
Also,
2
200
8 . 9
2
1
100
40
|
.
|

\
|
=
u

On solving s m u / 70 =

5.
x y
xv v or
dt
dx
x
dt
dy
x y 2 2 ;
2
| | | = = =
Or
( )
dt
v d
x
dt
dx
v
dt
dv
x
x
y
| | 2 2 + =
=
2
2 0 . 2
x x x
v v v | | = +

= , 0
dt
dv
x
because the particle has acceleration along Y-direction]
As per question,
2
2
x
x
v
dt
dv
| o = =
Or
|
o
2
=
x
v

6. Refer figure, horizontal component of velocity at A
QUESTION BANK
113


2 2
60 cos
0
ut
t u AC
u
u v
H H
= = = =

3 3
2
2
30 sec
0
t u t u
AC AB = = =

7. Here, u | u o = = , 2

Time of flight of A is,
( )
|
| o
cos
sin 2
1
g
u
T

=

( )
u
u
u u
tan
2
cos
2 sin 2
g
u
g
u
=

=
Time of flight of B is, u
u
u
tan
2
cos
sin 2
2
g
u
g
u
T = =
So T
1
= T
2
. The acceleration of both the particles is g downwards. Therefore, relative acceleration between the
two is zero or relative motion between the two is uniform. The relative velocity of A w.r.t. B is towards AB,
therefore collision will take place between the two in mid air.

8. Velocity along x-axis, 2 8 = = t
dt
dx
v
x

Or ( )dt t dx 2 8 =
Integrating it, we have
c t t c t
t
x + = + = 2 4 2
2
8
2
2

At c so x t + = = = 2 2 4 14 ; 14 , 2
2

Or 2 = c
QUESTION BANK
114
2 2 4
2
+ = t t x (i)
Also, dt dy or
dt
dy
v
y
2 2 = = =
Integrating it, ' 2 c t y + = .
At 0 ' ' 2 2 4 ; 4 , 2 = + = = = c or c so y t
t y 2 = .(ii)
In order to find the equation of path of projectile we have to eliminate t form (i) and (ii). From (ii)
2 / y t = . Putting it in (i), we get
2 2
2
2
4
4
2
2
+ = + = y y
y y
x

9. Taking horizontal motion from O to W, we have
t u t u or t u s
2
1
60 cos
2
3
0
= = =
Or
u
t
3
= (1)
Taking vertical motion from O to W, we have
m s If t g t u s 0 . 1
2
1
2
= =

2
0
3
10
2
1 3
60 sin 0 . 1
u u
u =

2 2
15
2
3 15
3
2
3
1
u u
= =
30 30
2
1
1
2
3 15
2
2
= = = = u or u or
u

If , 5 . 1 m s = then

2 2
0
15
2
3 3
10
2
1 3
60 sin 5 . 1
u u u
u = =
= = u or
u
0
15
2


10. If the force is acting on particle in the direction of motion, the particle will describe straight line path. If the
force is acting at an angle of motion of the particle, the path of a particle is parabolic path.
QUESTION BANK
115

11. Horizontal range will be the same when angle of projection is ( ) u u
0
90 or . The speed of projectile at the
point of projection is equal to the speed of landing in the absence of frectional forces or viscous forces.

12. When
0
90 = + | o , then horizontal range is same
Maximum Height
g
v
H
2
sin
2 2
u
=
And time of flight
g
v
T
u sin 2
=
i.e. u u sin sin
2
T and H

A B A B
T T and H H > >

13. The trajectory of projectile motion is
80 /
2
x x y =

14. Comparing it with the standard equation of projectile given angular projection with velocity u making an angle
u with the horizontal direction
i.e.
u
u
2
=
cos 2
tan
2
2
u
x g
x y
We have; 1 tan = u
Or ( ) 1 tan 45
1 0
= = u
And
80
1
cos 2
2 2
=
u u
g

Or
( )
800
2 / 1 2
10 80
cos 2
80
2 2
2
=

=
u
g
u
Or s m u / 2 20 =

15. Maximum height
g
u
2
sin
2 2
u
=

( )
m 20
10 2
45 sin 2 20
0 2
2
=

=

QUESTION BANK
116
16. Since after (two + one) second, the projectile is traveling horizontally, so
s T or
T
6 3 2 3 1 2
2
= = = + =

17. Time of flight,
g
u
T
y
2
=
Or s m
gT
u
y
/ 30
2
6 10
2
=

= =
Vertical velocity after 2 seconds, gt u v
y y
=
s m/ 10 2 10 30 = =
Horizontal velocity after 2 seconds,
x x
u v =
As per question,
x x
y
u v
v
10
30 tan
0
= =
Or s m u
x
/ 3 10
30 tan
10
0
= =
( ) ( )
2
2
2 2
30 3 10 + = + =
y x
u u u
s m/ 3 20 =

18.
0
60 tan 3
3 10
30
tan = = = =
x
y
u
u
u

0
60 = u

19. (A) Comparing the equation,
2
bx ax y = with the equation of projectile motion

u
u
2 2
2
cos 2
tan
u
x g
x y =
We have, b
u
g
and a = =
u
u
2 2
cos 2
tan
Or ( ) b
u
g
or b
u
g
= + = u u
2
2
2
2
tan 1
2
sec
2

Or ( )
2 / 1
2
1
2
(

+ = a
b
g
u
QUESTION BANK
117
(B) Horizontal range u u cos sin 2
2
g
u
=

b
a
g
u
= = u u tan cos
2
2
2

(C) Maximum height
( ) u
u u
2
2 2 2 2
tan
2
cos
2
sin
g
u
g
u
= =

b
a
g
u
4
tan
4
cos 2
2
2
2 2
= = = u
u

(D) As,
b
g
u so
u
g
b
2
cos ,
cos 2
2 2
= = u
u

Time of flight u
u u
tan
cos 2 sin 2
= =
g
u
g
u


gb
a a
b
g
g
2
2
2
= =

20. (A) Horizontal range,
g
u
R
u sin 2
=
It is maximum if the value of u 2 sin is maximum.
It is so when
0 0
60 30 or = u
(B) Maximum vertical height
g
u
2
sin 2
2
u
=
It is maximum if the value of u sin is maximum. It is so when
0
75 = u
(C) Time of flight
g
u u sin 2
=
It is minimum if the value of u sin is minimum, i.e.,
0
15 = u
(D) Horizontal range is minimum if u 2 sin has minimum value
It is so if
0 0
75 15 or = u

You might also like